nursing 110 final – Flashcards

Unlock all answers in this set

Unlock answers
question
The nurse is participating at a health fair at the local mall giving influenza vaccines to senior citizens. What level of prevention is the nurse practicing? A) Primary prevention B) Secondary prevention C) Tertiary prevention D) Quaternary prevention
answer
Primary prevention is aimed at health promotion and includes health-education programs, immunizations, and physical and nutritional fitness activities. It can be provided to an individual and includes activities that focus on maintaining or improving the general health of individuals, families, and communities. It also includes specific protection such as immunization for influenza.
question
A patient experienced a myocardial infarction 4 weeks ago and is currently participating in the daily cardiac rehabilitation sessions at the local fitness center. In what level of prevention is the patient participating? A) Primary prevention B) Secondary prevention C) Tertiary prevention D) Quaternary prevention
answer
Tertiary prevention involves minimizing the effects of long-term disease or disability by interventions directed at preventing complications and deterioration following the myocardial infarction. Tertiary-prevention activities are directed at rehabilitation rather than diagnosis and treatment. Care at this level aims to help patients achieve as high a level of functioning as possible, despite the limitations caused by illness or impairment. This level of care is called preventive care because it involves preventing further disability or reduced functioning.
question
Based on the transtheoretical model of change, what is the most appropriate response to a patient who states: "Me, exercise? I haven't done that since junior high gym class, and I hated it then!" A) "That's fine. Exercise is bad for you anyway." B) "OK. I want you to walk 3 miles 4 times a week, and I'll see you in 1 month." C) "I understand. Can you think of one reason why being more active would be helpful for you?" D) "I'd like you to ride your bike 3 times this week and eat at least four fruits and vegetables every day."
answer
The patient's response indicates that the patient is in the precontemplation stage and does not intend to change his behavior in the next 6 months. In this stage the patient is not interested in information about the behavior and may be defensive when confronted with it. Asking an open-ended question may stimulate the patient to identify a reason to begin a behavior change. Nurses are challenged to motivate and facilitate change in health behavior when working with individuals.
question
A patient comes to the local health clinic and states: "I've noticed how many people are out walking in my neighborhood. Is walking good for you?" What is the best response to help the patient through the stages of change for exercise? A) "Walking is OK. I really think running is better." B) "Yes, walking is great exercise. Do you think you could go for a 5-minute walk next week?" C) "Yes, I want you to begin walking. Walk for 30 minutes every day and start to eat more fruits and vegetables." D) "They probably aren't walking fast enough or far enough. You need to spend at least 45 minutes if you are going to do any good."
answer
The patient's response indicates that the patient is in the contemplative state, possibly intending to make a behavior change within the next 6 months. The nurse's statement reinforces the behavior and provides a specific goal for the patient to begin a walking plan.
question
A male patient has been laid off from his construction job and has many unpaid bills. He is going through a divorce from his marriage of 15 years and has been seeing his pastor to help him through this difficult time. He does not have a primary health care provider because he has never really been sick and his parents never took him to the physician when he was a child. Which external variables influence the patient's health practices? (Select all that apply.) A) Difficulty paying his bills B) Seeing his pastor as a means of support C) Family practice of not routinely seeing a health care provider D) Stress from the divorce and the loss of a job
answer
External factors impacting health practices include family beliefs and economic impact. How patients' families use health care services generally affects their health practices. Their perceptions of the serious nature of diseases and their history of preventive care behaviors (or lack of them) influence how patients will think about health. Economic variables may affect a patient's level of health by increasing the risk for disease and influencing how or at what point the patient enters the health care system.
question
The nurse is conducting a home visit with an older adult couple. She assesses that the lighting in the home is poor and there are throw rugs throughout the home and a low footstool in the living room. She discusses removing the rugs and footstool and improving the lighting with the couple. The nurse is addressing which level of need according to Maslow? A) Physiological B) Safety and security C) Love and belonging D) Self-actualization
answer
The teaching addresses the need for safety and security. The throw rugs, low lighting, and low stool are hazards that can cause falls in the elderly. Preventing falls is a priority safety issue for older adults.
question
When taking care of patients, the nurse routinely asks them if they take any vitamins or herbal medications, encourages family members to bring in music that the patient likes to help the patient relax, and frequently prays with her patients if that is important to them. The nurse is practicing which model? A) Holistic B) Health belief C) Transtheoretical D) Health promotion
answer
The nurse is using a holistic model of care that considers emotional and spiritual well-being and other dimensions of an individual to be important aspects of physical wellness. The holistic health model of nursing attempts to create conditions that promote optimal health. Nurses using the holistic nursing model recognize the natural healing abilities of the body and incorporate complementary and alternative interventions such as music therapy, reminiscence, relaxation therapy, therapeutic touch, and guided imagery because they are effective, economical, noninvasive, nonpharmacological complements to traditional medical care.
question
When illness occurs, different attitudes about it cause people to react in different ways. What do medical sociologists call this reaction to illness? A) Health belief B) Illness behavior C) Health promotion D) Illness prevention
answer
Illness behavior involves how people monitor their bodies, define and interpret their symptoms, take remedial actions, and use the resources in the health care system. Personal history, social situations, social norms, and past experiences can affect illness behavior.
question
A patient at the community clinic asks the nurse about health promotion activities that she can do because she is concerned about getting diabetes mellitus since her grandfather and father both have the disease. This statement reflects that the patient is in what stage of the health belief model? A) Perceived threat of the disease B) Likelihood of taking preventive health action C) Analysis of perceived benefits of preventive action D) Perceived susceptibility to the disease.
answer
The health belief model addresses the relationship between a person's beliefs and behaviors. It provides a way of understanding and predicting how patients will behave in relation to their health and how they will comply with health care therapies. In the perceived susceptibility to the disease phase, the patient recognizes the familial link to the disease.
question
A nurse works in a special care unit for children with severe immunology problems and is caring for a 3-year-old boy from Greece. The boy's father is with him while his mother and sister are back in Greece. The nurse is having difficulty communicating with the father. What action does the nurse take? A) Care for the boy as she would any other patient B) Ask the manager to talk with the father and keep him out of the unit C) Have another nurse care for the boy because maybe that nurse will do better with the father D) Search for help with interpretation and understanding of the cultural differences by contacting someone from the local Greek community
answer
The nurse needs to understand how the Greek culture impacts the father's health beliefs and communication with health care providers. Cultural variables must be incorporated into the child's plan of care. Cultural background influences beliefs, values, and customs. It influences the approach to the health care system, personal health practices, and the nurse-patient relationship. Cultural background may also influence an individual's beliefs about causes of illness and remedies or practices to restore health. If nurses are not aware of their own and other cultural patterns of behavior and language, they may not be able to recognize and understand a patient's behavior and beliefs and may have difficulty interacting with the patient.
question
A patient with a 20-year history of diabetes mellitus had a lower leg amputation. Which statement made by the patient indicates that he is experiencing a problem with body image? A) "I just don't have any energy to get out of bed in the morning." B) "I've been attending church regularly with my wife since I got out of the hospital." C) "My wife has taken over paying the bills since I've been in the hospital." D) "I don't go out very much because everyone stares at me."
answer
The amputation resulted in a change in physical appearance that caused a change in body image. Reactions of patients and families to changes in body image depend on the type of changes (e.g., loss of a limb or an organ), their adaptive capacity, the rate at which changes take place, and the support services available. When a change in body image such as results from a leg amputation occurs, the patient generally adjusts in the following phases: shock, withdrawal, acknowledgment, acceptance, and rehabilitation. The patient's statement indicates he is in the stage of withdrawal.
question
The patient states she joined a fitness club and attends the aerobics class three nights a week. The patient is in what stage of behavioral change? A) Precontemplation B) Contemplation C) Preparation D) Action
answer
The patient is in the action stage of behavioral change. In this stage the patient is actively engaged in strategies to change behavior. This stage may last up to 6 months.
question
The nurse is developing a health promotion program on healthy eating and exercise for high school students using the health belief model as a framework. Which statement made by a nursing student is related to the individual's perception of susceptibility to an illness? A) "I don't have time to exercise because I have to work after school every night." B) "I'm worried about becoming overweight and getting diabetes because my father has diabetes." C) "The statistics of how many teenagers are overweight is scary." D) "I've decided to start a walking club at school for interested students."
answer
The statement indicates that the patient is concerned about developing diabetes and believes that there is a risk or susceptibility based on recognition of a familial link for the disease. Once this link is recognized, the patient may perceive the personal risk for diabetes.
question
The nurse assesses the following risk factors for coronary artery disease (CAD) in a male patient. Which factors are classified as genetic and physiological? (Select all that apply.) A) Sedentary lifestyle B) Father died from CAD at age 50 C) History of hypertension D) Eats diet high in sodium E) Elevated cholesterol level F) Age is 44 years
answer
Genetic and physiological risk factors include those related to heredity, genetic predisposition to an illness, or those that involve the physical functioning of the body. Certain physical conditions such as being pregnant or overweight place increased stress on physiological systems (e.g., the circulatory system), increasing susceptibility to illness in these areas. A person with a family history of coronary artery disease is at risk for developing the disease later in life because of a hereditary and genetic predisposition to the disease.
question
Which activity represents secondary prevention? A) A home health care nurse visits a patient's home to change a wound dressing. B) A 50-year-old woman with no history of disease attends the local health fair and has her blood pressure checked. C) The school health nurse provides a program to the first-year students on healthy eating. D) The patient attends cardiac rehabilitation sessions weekly.
answer
Secondary prevention focuses on individuals who are experiencing health problems or illnesses and who are at risk for developing complications or worsening conditions. The home health nurse changing the wound dressing is an activity that is focused on preventing complications. Much of the nursing care related to secondary prevention is delivered in homes, hospitals, or skilled nursing facilities.
question
A nurse hears a colleague tell a nursing student that she never touches a patient unless she is performing a procedure or doing an assessment. The nurse tells the student that from a caring perspective: A) She does not touch the patients either. B) Touch is a type of verbal communication. C) There is never a problem with using touch. D) Touch forms a connection between nurse and patient.
answer
D.Touch is relational and leads to a connection between nurse and patient. It involves contact and noncontact touch. Contact touch involves obvious skin-to-skin contact, whereas noncontact touch refers to eye contact.
question
Of the five caring processes described by Swanson, which describes "knowing the patient"? A) Anticipating the patient's cultural preferences B) Determining the patient's physician preference C) Establishing an understanding of a specific patient D) Gathering task-oriented information during assessment
answer
C.Knowing the context of a patient's illness helps you choose and individualize interventions that will actually help him or her. Strive to understand an event as it has meaning in the life of the other. Knowing the patient is essential when providing patient-centered care. Two elements that facilitate knowing are continuity of care and clinical expertise.
question
A Muslim woman enters the clinic to have a woman's health examination for the first time. Which nursing behavior applies Swanson's caring process of "knowing the patient?" A) Sharing feelings about the importance of having regular woman's health examinations B) Gaining an understanding of what a woman's health examination means to the patient C) Recognizing that the patient is modest; obtaining gendercongruent caregiver D) Explaining the risk factors for cervical cancer
answer
B. You should strive to understand an event as it has meaning in the life of the other. Knowing the patient is essential when providing patient-centered care.
question
Helping a new mother through the birthing experience demonstrates which of Swanson's five caring processes? A) Knowing B) Enabling C) Doing for D) Being with
answer
B. The caring behavior of enabling facilitates the other's passage through life transitions (e.g., birth, death) and unfamiliar events. When a nurse practices enabling, the patient and nurse work together to identify alternatives and resources.
question
A patient is fearful of upcoming surgery and a possible cancer diagnosis. He discusses his love for the Bible with his nurse, who recommends a favorite Bible verse. Another nurse tells the patient's nurse that there is no place in nursing for spiritual caring. The patient's nurse replies: A) "Spiritual care should be left to a professional." B) "You are correct, religion is a personal decision." C) "Nurses should not force their religious beliefs on patients." D) "Spiritual, mind, and body connections can affect health."
answer
D. Spirituality offers a sense of connectedness, intrapersonally (connected with oneself), interpersonally (connected with others and the environment), and transpersonally (connected with the unseen, God, or a higher power). In a caring relationship the patient and nurse come to know one another so both move toward a healing relationship.
question
Which of the following is a strategy for creating work environments that enable nurses to demonstrate more caring behaviors? A) Increasing the working hours of the staff B) Increasing salary benefits of the staff C) Creating a setting that allows flexibility and autonomy for staff D) Encouraging increased input concerning nursing functions from physicians
answer
C.These factors all affect nursing satisfaction. When nurses' job satisfaction is high, they have a greater connectedness with their patients and believe that caring practices are part of the nursing culture.
question
When a nurse helps a patient find the meaning of cancer by supporting beliefs about life, this is an example of: A) Instilling hope and faith. B) Forming a human-altruistic value system. C) Cultural caring. D) Being with.
answer
A. Instilling hope and faith helps to increase an individual's capacity to get through an event or transition and face the future with meaning.
question
An example of a nurse caring behavior that families of acutely ill patients perceive as important to patients' well-being is: A) Making health care decisions for patients. B) Having family members provide a patient's total personal hygiene. C) Injecting the nurse's perceptions about the level of care provided. D) Asking permission before performing a procedure on a patient.
answer
D. Caring for the family takes into consideration the context of the patient's illness and the stress it imposes on all members.
question
A nurse demonstrates caring by helping family members: A) Become active participants in care. B) Provide activities of daily living (ADLs). C) Remove themselves from personal care. D) Make health care decisions for the patient.
answer
A. Caring for the family takes into consideration the context of the patient's illness and the stress it imposes on all members.
question
Listening is not only "taking in" what a patient says; it also includes: A) Incorporating the views of the physician. B) Correcting any errors in the patient's understanding. C) Injecting the nurse's personal views and statements. D) Interpreting and understanding what the patient means.
answer
D. Listening is powerful. It conveys the nurse's full attention and interest. A true caring presence involves listening. Listen to what is important to another person and the meaning of a situation to that person.
question
A nurse is caring for an older adult who needs to enter an assisted-living facility following discharge from the hospital. Which of the following is an example of listening that displays caring? A) The nurse encourages the patient to talk about his concerns while reviewing the computer screen in the room. B) The nurse sits at the patient's bedside, listens as he relays his fear of never seeing his home again, and then asks if he wants anything to eat. C) The nurse listens to the patient's story while sitting on the side of the bed and then summarizes the story. D) The nurse listens to the patient talk about his fears of not returning home and then tells him to think positively.
answer
C. Attentive listening lets the nurse hear the patient's story and then correctly summarize it. It does not occur when the nurse is distracted by equipment or other personnel. The importance of listening is not to distract the patient or solve the problem, but rather to hear what the patient has to say and understand what the situation means to him.
question
Presence involves a person-to-person encounter that: A) Enables patients to care for self. B) Provides personal care to a patient. C) Conveys a closeness and a sense of caring. D) Describes being in close contact with a patient.
answer
C. Providing presence is a person-to-person encounter conveying closeness and a sense of caring. It involves "being there" and "being with." "Being there" is not only a physical presence but also includes communication and understanding. Presence is an interpersonal process that is characterized by sensitivity, holism, intimacy, vulnerability, and adaptation to unique circumstances.
question
A nurse enters a patient's room, arranges the supplies for a Foley catheter insertion, and explains the procedure to the patient. She tells the patient what to expect; just before inserting the catheter, she tells the patient to relax and that, once the catheter is in place, she will not feel the bladder pressure. The nurse then proceeds to skillfully insert the Foley catheter. This is an example of what type of touch? A) Caring touch B) Protective touch C) Task-oriented touch D) Interpersonal touch
answer
C. Nurses use task-orientated touch when performing a task or procedure. An expert nurse learns that any procedure is more effective when administered carefully and in consideration of any patient concern.
question
A hospice nurse sits at the bedside of a male patient in the final stages of cancer. He and his parents made the decision that he would move home and they would help him in the final stages of his disease. The family participates in his care, but lately the nurse has increased the amount of time she spends with the family. Whenever she enters the room or approaches the patient to give care, she touches his shoulder and tells him that she is present. This is an example of what type of touch? A) Caring touch B) Protective touch C) Task-oriented touch D) Interpersonal touch
answer
A. Caring touch is a form of nonverbal communication. You express this in the way you hold a patient's hand, give a back massage, gently position a patient, or participate in a conversation. When using a caring touch, you connect with the patient physically and emotionally.
question
Match the following caring behaviors with their definitions. A. Sustaining faith in one's capacity to get through a situation B. Striving to understand an event's meaning for another person C. Being emotionally there for another person D. Providing for another as he or she would do for themselves. Knowing Being with Doing for Maintaining belief
answer
Striving to understand an event's meaning for another person:Knowing, Being emotionally there for another person:Being with, Providing for another as he or she would do for themselves.:Doing for, Sustaining faith in one's capacity to get through a situation:Maintaining belief
question
The nurse's first action after discovering an electrical fire in a patient's room is to: A) Activate the fire alarm. B) Confine the fire by closing all doors and windows. C) Remove all patients in immediate danger. D) Extinguish the fire by using the nearest fire extinguisher.
answer
C. Follow the acronym RACE. The first step, R, is to rescue and remove all patients in immediate danger
question
A parent calls the pediatrician's office frantic about the bottle of cleaner that her 2-year-old son drank. Which of the following is the most important instruction the nurse gives to this parent? A) Give the child milk. B) Give the child syrup of ipecac. C) Call the poison control center. D) Take the child to the emergency department.
answer
C. A poison control center is the best resource for patients and parents needing information about the treatment of an accidental poisoning
question
The nursing assessment on a 78-year-old woman reveals shuffling gait, decreased balance, and instability. On the basis of the patient's data, which one of the following nursing diagnoses indicates an understanding of the assessment findings? A) Activity intolerance B) Impaired bed mobility C) Acute pain D) Risk for falls
answer
D. For adults age 65 and older, impaired balance and difficulty with gait are risks for the nursing diagnosis of risk for falls
question
A couple is with their adolescent daughter for a school physical and state they are worried about all the safety risks affecting this age. What is the greatest risk for injury for an adolescent? A) Home accidents B) Physiological changes of aging C) Poisoning and child abduction D) Automobile accidents, suicide, and substance abuse
answer
D. Risks to the safety of adolescents involve many factors outside the home because much of their time is spent away from home and with their peer group. According to the Centers for Disease Control and Prevention, the risk of motor vehicle accidents is higher among 16- to 19-year-old drivers than any other age-group. In an attempt to relieve the tensions associated with physical and psychosocial changes and peer pressures, some adolescents engage in risk-taking behaviors such as smoking, drinking alcohol, and using drugs.
question
The nurse found a 68-year-old female patient wandering in the hall. The patient says she is looking for the bathroom. Which interventions are appropriate to ensure the safety of the patient? (Select all that apply.) A) Insert a urinary catheter. B) Leave a night light on in the bathroom. C) Ask the physician to order a restraint. D) Keep the bed in low position with upper and lower side rails up. E) Assign a staff member to stay with the patient. F) Provide scheduled toileting during the night shift. G) Keep the pathway from the bed to the bathroom clear.
answer
B,F,G. Older adults in an unfamiliar environment may become confused. A night light may be beneficial for safety and orientation. Toileting is a common reason for a patient attempting to get out of bed. Placing the patient on a routine toileting schedule should help decrease this risk factor. Hospital environments can quickly become cluttered with equipment, personal items, and other things that create a hazard for falling. Keep pathways clear. All alternatives should be tried and considered before using a restraint. Restraint should not be an initial response. The bed should be kept in a low position. Upper side rails may be used; however, the addition of lower side rails can increase the risk of injury. The use of side rails alone for a disoriented patient may cause more confusion and further injury. A confused patient who is determined to get out of bed attempts to climb over the side rail or climbs out at the foot of the bed. Either attempt usually results in a fall or injury.
question
The family of a patient who is confused and ambulatory insists that all four side rails be up when the patient is alone. What is the best action to take in this situation? (Select all that apply.) A) Contact the nursing supervisor. B) Restrict the family's visiting privileges. C) Ask the family to stay with the patient. D) Inform the family of the risks associated with side-rail use. E) Thank the family for being conscientious and put the four rails up. F) Discuss alternatives with the family that are appropriate for this patient.
answer
C,D,F. The family is concerned about ensuring a safe environment for their loved one. The nurse should discuss their concerns, the risk of using restraints related to using four side rails, and safer alternatives such as the presences of a family member. If the family still insists on use of four side rails, you could contact the nursing supervisor to further discuss the situation with them. This is not a reason to restrict visitation; but, although you should appreciate their concern, the use of four side rails should be avoided.
question
A physician writes an order to apply a wrist restraint to a patient who has been pulling out a surgical wound drain. Place the following steps for applying the restraint in the correct order. ___ 1. Explain what you plan to do. ___ 2. Wrap a limb restraint around wrist or ankle with soft part toward skin and secure. ___ 3. Determine that restraint alternatives fail to ensure patient's safety. ___ 4. Identify the patient using proper identifier. ___ 5. Pad the patient's wrist.
answer
3,4,1,5,2.
question
A child in the hospital starts to have a grand mal seizure while playing in the playroom. What is your most important nursing intervention during this situation? A) Begin cardiopulmonary respiration. B) Restrain the child to prevent injury. C) Place a tongue blade over the tongue to prevent aspiration. D) Clear the area around the child to protect the child from injury.
answer
D. Once a seizure begins, you need to monitor the patient and provide a safe environment. A seizure is not an indication for cardiopulmonary resuscitation. A person having a seizure should not be restrained, but the environment should be made safe. Objects should not be forced into the mouth. See the Skills in the chapter for more information.
question
A 62-year-old woman is being discharged home with her husband after surgery for a hip fracture from a fall at home. When providing discharge teaching about home safety to this patient and her husband, the nurse knows that: A) A safe environment promotes patient activity. B) Assessment focuses on environmental factors only. C) Teaching home safety is difficult to do in the hospital setting. D) Most accidents in the older adult are caused by lifestyle factors.
answer
A. Older adults are frequently fearful of falling and thus often limit activity. A safe environment, which decreases the risk of a fall, promotes patient activity.
question
A fragile, 87-year-old nursing home resident is admitted to the hospital with dehydration and increased confusion. The patient has upper limb restraints to prevent her from pulling out her nasogastric tube. What instructions does the nurse give to nursing assistive personnel (NAP)?
answer
The use of restraints is associated with serious complications resulting from immobilization such as pressure ulcers, pneumonia, constipation, and incontinence. In some cases death has resulted because of restricted breathing and circulation. The restraint itself could injure the underlying skin. Routine checks are required to prevent or decrease these complications. The NAP needs to notify the nurse if there is a change in skin integrity, circulation, or patient's breathing and provide range of motion, nutrition and hydration, skin care, toileting, and opportunities for socialization at least every 2 hours.
question
The nursing assessment of an 80-year-old patient who demonstrates some confusion but no anxiety reveals that the patient is a fall risk because she continues to get out of bed without help despite frequent reminders. The initial nursing intervention to prevent falls for this patient is to: A) Place a bed alarm device on the bed. B) Place the patient in a belt restraint. C) Provide one-on-one observation of the patient. D) Apply wrist restraints.
answer
A. Consider and implement alternatives as appropriate before the use of a restraint. A bed alarm is an alternative that the nurse implements independently.
question
To ensure the safe use of oxygen in the home by a patient, which of the following teaching points does the nurse include? (Select all that apply.) A) Smoking is prohibited around oxygen. B) Demonstrate how to adjust the oxygen flow rate based on patient symptoms. C) Do not use electrical equipment around oxygen. D) Special precautions may be required when traveling with oxygen
answer
A,C,D. When oxygen is in use, precautions need to be taken to prevent fire and protect the patient. Patients need to be taught precautions, which include posting "Oxygen in Use" signage, not using oxygen around electrical equipment or flammable products, properly handling oxygen cylinders/containers, ensuring that tubing is unobstructed, not adjusting liter flow without a physician's order, and taking precautions when traveling with oxygen
question
How does the nurse support a culture of safety? (Select all that apply.) A) Completing incident reports when appropriate B) Completing incident reports for a near miss C) Communicating product concerns to an immediate supervisor D) Identifying the person responsible for an incident
answer
A,B,C. Completing incident reports for actual and near-miss events helps the facility track information and identify trends and patterns that need to be addressed. Communicating product concerns to a responsible supervisor allows the facility to further investigate and determine if additional action is required.
question
You are admitting Mr. Jones, a 64-year-old patient who had a right hemisphere stroke and a recent fall. The wife stated that he has a history of high blood pressure, which is controlled by an antihypertensive and a diuretic. Currently he exhibits left sided neglect and problems with spatial and perceptual abilities and is impulsive. He has moderate left-sided weakness that requires the assistance of two and the use of a gait belt to transfer to a chair. He currently has an intravenous (IV) line and a urinary catheter in place. What factors increase his fall risk at this time? (Select all that apply.) A) Smokes a pack a day B) Used a cane to walk at home C) Takes antihypertensive and diuretics D) History of recent fall E) Neglect, spatial and perceptual abilities, impulsive F) Requires assistance with activity, unsteady gait G) IV line, urinary catheter
answer
C,D,E,F,G. Smoking is not a risk factor for falls. Because the patient used the cane at home, it is not a current risk factor for falls. Risk is determined by his current status.
question
At 3 am the emergency department nurse hears that a tornado hit the east side of town. What action does the nurse take first? A) Prepare for an influx of patients B) Contact the American Red Cross C) Determine how to restore essential services D) Evacuate patients per the disaster plan
answer
A. The emergency department nurse needs to prepare for the potential influx of patients first. Staff need to be aware of the disaster plan. Patients may need to be evaluated but not initially. The American Red Cross is not contacted initially. Determination of how to restore essential services is part of the disaster plan and is determined before an actual event.
question
The Collins family includes a mother, Jean; stepfather, Adam; two teenage biological daughters of the mother, Lisa and Laura; and a biological daughter of the father, 25-year-old Stacey. Stacey just moved home following the loss of her job in another city. The family is converting a study into Stacey's bedroom and is in the process of distributing household chores. When you talk to members of the family, they all think that their family can adjust to lifestyle changes. This is an example of family: A) Diversity. B) Durability. C) Resiliency. D) Configuration.
answer
C. Resiliency is the ability of the family to cope with the unexpected. In this scenario the family used resources to provide some short-term solutions for the adult child's return home.
question
The most common reason grandparents are called on to raise their grandchildren is because of: A) Single parenthood. B) Legal interventions. C) Dual-income families. D) Increased divorce rate.
answer
B. This new parenting responsibility is caused by a number of societal factors: the increase in the divorce rate, dual-income families, and single parenthood. But most often it is a consequence of legal intervention when parents are unfit or renounce their parental obligations.
question
A family's access to adequate health care, opportunity for education, sound nutrition, and decreased stress is affected by: A) Development. B) Family function. C) Family structure. D) Economic stability.
answer
D. The ability of families to meet health care, education, and basic needs is often affected by the economic resources of the family
question
David Singer is a single parent of a 3-year-old boy, Kevin. Kevin has well-managed asthma and misses day care infrequently. David is in school studying to be an information technology professional. His income and time are limited, and he admits to going to fast-food restaurants frequently for dinner. However, he and his son spend a lot of time together. David receives state-supported health care for his son, but he does not have health insurance or a personal physician. He has his son enrolled in a government-assisted day care program. Which of the following are risks to this family's level of health? (Select all that apply.) A) Economic status B) Chronic illness C) Underinsured D) Government-assisted day care
answer
A,C. David's economic status is stretched. He has multiple resources for his son, but he is not insured. Thus, as a result, there is a potential that David does not follow through with personal health promotion activities. Although asthma is a chronic illness, this is well managed, and there is adequate health care for his son.
question
The Cleric family, which includes a mother, stepfather, two teenage biological daughters of the mother, and a biological daughter of the father is an example of a(n): A) Nuclear family. B) Blended family. C) Extended family. D) Alternative family
answer
B. Blended families result when two people who have children from a previous marriage/relationship marry.
question
Which of the following are possible outcomes with clear family communication? (Select all that apply.) A) Family goals B) Decision making C) Methods of discipline D) Impaired coping
answer
A,B,C. Clear and direct family communication assists the family in creating goals, decision making, progressing through the family development cycle, and coping with stressors.
question
Communication among family members is an example of family: A) Attributes. B) Function. C) Structure. D) Development
answer
B. Communication is a component of family functioning, whether that be setting goals, coping, or establishing discipline. Family functioning is what the family does, and communication is an important component of function
question
Which of the following contribute to family hardiness? (Select all that apply.) A) Family meetings B) Established family roles C) Willingness to change in time of stress D) Passive orientation to life
answer
A,B,C. Family hardiness is the internal strengths and durability of the family unit. It includes a sense of control over the outcome of life, a view of change as beneficial and growth producing, and an active orientation (such as family meetings) rather than passive orientation in adapting to stressful events. Family meetings, understanding of roles, and adaptation to stressors along with a willingness to change affect family hardiness.
question
Which of the following demonstrate family resiliency? (Select all that apply.) A) Resuming full-time work when spouse loses job B) Arguing ways to deal with problems among siblings C) Developing hobbies when children leave home D) Placing blame on family members
answer
A,C. Family resiliency is the ability to cope with expected and unexpected stressors. Resiliency helps to evaluate healthy responses when individuals and families are experiencing stressful events.
question
When nurses view the family as context, their primary focus is on the: A) Family members within a system. B) Family process and relationships. C) Family relational and transactional concepts. D) Health needs of an individual member.
answer
D. When you view the family as context, the primary focus is on the health and development of an individual member existing within a specific environment (i.e., the patient's family). Although the focus is on the individual's health status, assess how much the family provides the individual's basic needs.
question
Diane is a hospice nurse who is caring for the Robinson family. This family is providing end-of-life care for their grandmother, who has terminal breast cancer. When Diane visits the home 3 times a week, she focuses on symptom management for the grandmother and assists the family with coping skills. Diane's approach is an example of which of the following? A) Family as context B) Family as patient C) Family as system D) Family as structure
answer
B. When the family as patient is the approach, family processes and relationships (e.g., parenting or family caregiving) are the primary focuses of nursing care.
question
Which of the following are included in a family function assessment? (Select all that apply.) A) Cultural practices B) Decision making C) Rituals and celebrations D) Neighborhood crime data
answer
A,B,C. Cultural practices help identify culturally related health practices, diets, and religious practices. Decision making provides information as to how the family copes and meets challenges related to changes in family life or dynamics. Rituals and celebrations address how a family celebrates accomplishments and how they deal with challenges. Neighborhood crime data are relevant for community assessment, but they do not give sufficient information about family function.
question
Karen Johnson is a single mother of a school-age daughter. Linda Brown is also a single mother of two teenage daughters. Karen and Linda are active professionals, have busy social lives, and date occasionally. Three years ago they decided to share a house and housing costs, living expenses, and child care responsibilities. The children consider one another as their family. This family form is considered a(n): A) Diverse family relationship. B) Blended family relationship. C) Extended family relationship. D) Alternative family relationship.
answer
D. This relationship includes multiadult households, "skip-generation" families, communal groups with children, "nonfamilies," cohabitating partners, and homosexual couples.
question
During a visit to a family clinic the nurse teaches the mother about immunizations, car seat use, and home safety for an infant and toddler. Which type of nursing interventions are these? A) Health promotion activities B) Acute care activities C) Restorative care activities D) Growth and development-care activities
answer
A. Health promotion activities focus on interventions designed to maintain the physical, social, emotional, and spiritual health of the family unit. They can include information about specific health behaviors, family coping techniques, and growth and development.
question
Which best defines family caregiving? (Select all that apply.) A) Designing a nurturing family to raise children B) Providing physical and emotional care for a family member C) Establishing a safe physical environment for a family D) Monitoring for side effects of illness and treatments
answer
B,C,D. Family caregiving involves the routine provision of services and personal care activities for a family member by spouses, siblings, or parents. Caregiving activities include finding resources, personal care (bathing, feeding, or grooming), monitoring for complications or side effects of illness and treatments, providing instrumental activities of daily living (shopping or housekeeping), and the ongoing emotional support and decision making that is necessary.
question
The nurse is aware that preschoolers often display a developmental characteristic that makes them treat dolls or stuffed animals as if they have thoughts and feelings. This is an example of: A) Logical reasoning. B) Egocentrism. C) Concrete thinking. D) Animism.
answer
D. This is the belief that inanimate objects have lifelike qualities; it is a component of magical thinking evident in preoperational thought
question
An 18-month-old child is noted by the parents to be "angry" about any change in routine. This child's temperament is most likely to be described as: A) Slow to warm up. B) Difficult. C) Hyperactive. D) Easy.
answer
B. Children described as "difficult" adapt slowly to new routines and express their emotions forcefully; they like consistent structure.
question
Nine-year-old Brian has a difficult time making friends at school and being chosen to play on the team. He also has trouble completing his homework and, as a result, receives little positive feedback from his parents or teacher. According to Erikson's theory, failure at this stage of development results in: A) A sense of guilt. B) A poor sense of self. C) Feelings of inferiority. D) Mistrust.
answer
C. School-age children need to feel real accomplishment and be accepted by peers to develop a sense of industry.
question
The nurse teaches parents how to have their children learn impulse control and cooperative behaviors. This would be during which of Erikson's stages of development? A) Trust versus mistrust B) Initiative versus guilt C) Industry versus inferiority D) Autonomy versus sense of shame and doubt
answer
B. Toddlers are learning that parents and society have expectations about behaviors and that they must learn to control their behavior.
question
When Ryan was 3 months old, he had a toy train; when his view of the train was blocked, he did not search for it. Now that he is 9 months old, he looks for it, reflecting the presence of: A) Object permanence. B) Sensorimotor play. C) Schemata. D) Magical thinking.
answer
A. He is now in Piaget's later stage of sensorimotor thought and has learned that objects exist even though he cannot see or touch them.
question
When preparing a 4-year-old child for a procedure, which method is developmentally most appropriate for the nurse to use? A) Allowing the child to watch another child undergoing the same procedure B) Showing the child pictures of what he or she will experience C) Talking to the child in simple terms about what will happen D) Preparing the child through play with a doll and toy medical equipment
answer
D. Preschoolers are in the preoperational stage of cognitive development and learn more easily when play is used to teach
question
A 35-year-old woman is speaking with you about her recent diagnosis of a chronic illness. She is concerned about her treatment options in relation to her ability to continue to care for her family. As she considers the options and alternatives, she incorporates information, her values, and emotions to decide which plan will be the best fit for her. She is using which form of cognitive development? A) Conventional reasoning B) Formal operations C) Integrity versus despair D) Postformal thought
answer
D. Adults recognize that there are various solutions to problems and that different situations demand different solutions.
question
You are caring for a recently retired man who appears withdrawn and says he is "bored with life." Applying the work of Havinghurst, you would help this individual find meaning in life by: A) Encouraging him to explore new roles. B) Encouraging relocation to a new city. C) Explaining the need to simplify life. D) Encouraging him to adopt a new pet.
answer
A. The activity theory states that continuing an active, involved lifestyle results in greater satisfaction and well-being.
question
Place the following stages of Freud's psychosexual development in the proper order by age progression. 1. Oedipal 2. Latency 3. Oral 4. Genital 5. Anal
answer
3, 5, 2, 1, 4
question
According to Piaget's cognitive theory, a 12-year-old child is most likely to engage in which of the following activities? A) Using building blocks to determine how houses are constructed B) Writing a story about a clown who wants to leave the circus C) Drawing pictures of a family using stick figures D) Writing an essay about patriotism
answer
B. As adolescents mature, their thinking moves to abstract and theoretical subjects. They have the capacity to reason with respect to possibilities.
question
Allison, age 15 years, calls her best friend Laura and is crying. She has a date with John, someone she has been hoping to date for months, but now she has a pimple on her forehead. Laura firmly believes that John and everyone else will notice the blemish right away. This is an example of the: A) Imaginary audience. B) False-belief syndrome. C) Personal fable. D) Personal absorption syndrome.
answer
A. Adolescents are quite egocentric and have the belief that everyone is focused on them and sees all of their flaws.
question
Elizabeth, who is having unprotected sex with her boyfriend, comments to her friends, "Did you hear about Kathy? You know, she fools around so much; I heard she was pregnant. That would never happen to me!" This is an example of adolescent: A) Imaginary audience. B) False-belief syndrome. C) Personal fable. D) Sense of invulnerability
answer
D. Adolescents can be risk takers and believe that they are immune to the negative consequences of behaviors; they are just beginning to be future oriented in their thought process and see everything as black or white
question
Teaching an older adult how to use e-mail to communicate with a grandchild who lives in another state is an example of ____________, which aids cognitive performance by using new approaches. A) Cognitive development B) Activity theory C) Selective optimization with compensation D) Formal operations
answer
C. As adults age, they put more of their energies into activities associated with meaningful relationships
question
Dave reports being happy and satisfied with his life. What do we know about Dave? A) He is in one of the later developmental periods, concerned with reviewing his life. B) He is atypical, since most people in any of the developmental stages report significant dissatisfaction with their lives. C) He is in one of the earlier developmental periods, concerned with establishing a career and satisfying long-term relationships. D) It is difficult to determine Dave's developmental stage since most people report overall satisfaction with their lives in all stages
answer
D. Each of the life stages can be achieved successfully and result in satisfaction, including old age
question
You are working in a clinic that provides services for homeless people. The current local regulations prohibit providing a service that you believe is needed by your patients. You adhere to the regulations but at the same time are involved in influencing authorities to change the regulation. This action represents which stage of moral development? A) Instrumental relativist orientation B) Social contract orientation C) Society-maintaining orientation D) Universal ethical principle orientation
answer
B. At this stage the individual recognizes that at times the law must be changed to meet the needs of society and that all people have basic rights, regardless of their social group.
question
In an interview with a pregnant patient, the nurse discussed the three risk factors that have been cited as having a possible effect on prenatal development. They are: A) Nutrition, stress, and mother's age. B) Prematurity, stress, and mother's age. C) Nutrition, mother's age, and fetal infections. D) Fetal infections, prematurity, and placenta previa.
answer
A. The woman's diet before and during pregnancy has a significant effect on fetal development; the mother's age may contribute to a risk for chromosomal defects (older mothers) or the lack of prenatal care (adolescent mothers); pregnancy is often accompanied by stress because of all of the developmental changes, and it is important to know whether or not the mother has an effective support system.
question
A parent has brought her 6-month-old infant in for a well-child check. Which of her statements indicates a need for further teaching? A) "I can start giving her whole milk at about 12 months." B) "I can continue to breastfeed for another 6 months." C) "I've started giving her plenty of fruit juice as a way to increase her vitamin intake." D) "I can start giving her solid food now."
answer
C. Breast milk or formula is recommended at this time; fruit juice is not considered a nutritive addition.
question
The type of injury a child is most vulnerable to at a specific age is most closely related to which of the following? A) Provision of adult supervision. B) Educational level of the parent C) Physical health of the child D) Developmental level of the child
answer
D. The child's cognitive and physical development need to be considered initially when assessing the potential risk for injury.
question
Which approach would be best for the nurse to use with a hospitalized toddler? A) Always give several choices. B) Set few limits to allow for open expression. C) Use noninvasive methods when possible. D) Gain cooperation before attempting treatment.
answer
D. Toddlers are learning to become independent and frequently display negative behavior if an effort to gain their trust is not provided initially. Providing too many choices does not support their efforts to gain control.
question
The nurse is providing information on prevention of sudden infant death syndrome (SIDS) to the mother of a young infant. Which of the following statements indicates that the mother has a good understanding? A) "I won't use a pacifier to help my baby sleep." B) "I'll be sure my baby does not spend any time on her abdomen." C) "I'll place my baby on her back for sleep." D) "I'll be sure to keep my baby's room cold."
answer
C,D. The American Academy of Pediatrics has clearly recommended that infants be placed on their backs for sleep to help prevent SIDS. Keeping the room cool is also important.
question
In evaluating the gross-motor development of a 5-month-old infant, which of the following would the nurse expect the infant to do? A) Roll from abdomen to back B) Move from prone to sitting unassisted C) Sit upright without support D) Turn completely over
answer
A. The 5-month-old infant should be able to turn from abdomen to back.
question
Parents are concerned about their toddler's negativism and ask the nurse for guidance. Which is the most appropriate recommendation? A) Provide more attention. B) Reduce opportunities for a "no" answer. C) Be consistent with punishment. D) Provide opportunities for the toddler to make decisions.
answer
B. Giving toddlers realistic choices reduces the opportunity for a negative response and helps support their need for control.
question
When nurses are communicating with adolescents, they should: A) Be alert to clues to their emotional state. B) Ask closed-ended questions to get straight answers. C) Avoid looking for meaning behind adolescents' words or actions. D) Avoid discussing sensitive issues such sex and drugs.
answer
A. Adolescents are searching for their identity and trying to become emotionally independent from parents while maintaining family ties. Depression, substance abuse, and violence are all real concerns during this period; thus the nurse must be aware of the adolescent's emotional state.
question
Which of the following statements is most descriptive of the psychosocial development of school-age children? A) Boys and girls play equally with each other. B) Peer influence is not yet an important factor to the child. C) They like to play games with rigid rules. D) Children frequently have "best friends."
answer
D. Peer relationships become very important to school-age children, and they usually develop close friendships.
question
You are caring for a 4-year-old child who is hospitalized for an infection. He tells you that he is sick because he was "bad." Which is the most correct interpretation of his comment? A) Indicative of extreme stress B) Representative of his cognitive development C) Suggestive of excessive discipline at home D) Indicative of his developing sense of inferiority
answer
B. Preschoolers exhibit "egocentric" thought, meaning that they truly believe that their thinking is shared by others and that they can control their environment by their thoughts.
question
At a well-child examination, the mother comments that her toddler eats little at mealtime, will only sit briefly at the table, and wants snacks all the time. Which of the following should the nurse recommend? A) Provide nutritious snacks. B) Offer rewards for eating at mealtimes. C) Avoid snacks so she is hungry at mealtime. D) Explain to her firmly why eating at mealtime is important.
answer
A. Toddlers are not growing as quickly as they did during infancy and thus eat smaller meals; nutritious snacks can help to ensure that they gain the nutrients they need.
question
An 8-year-old child is being admitted to the hospital from the emergency department with an injury from falling off her bicycle. Which of the following will most help her adjust to the hospital? A) Explain hospital routines such as meal times to her. B) Use terms such as "honey" and "dear" to show a caring attitude. C) Explain when her parents can visit and why siblings cannot come to see her. D) Since she is young, orient her parents to her room and hospital facility.
answer
A. School-age children are able to think logically and can classify objects or routines; having an understanding of what to expect can help them cope with a new experience.
question
The school nurse is counseling an obese 10-year-old child. What factor would be important to consider when planning an intervention to support the child's health? A) Concentrate on the child only rather than the family since it is the child's responsibility. B) Consider the use of medications to suppress the appetite. C) First plan for weight loss through dieting and then add activity as tolerated. D) Plan food intake to allow for growth
answer
D. Although growth slows down during the school-age years, it is still important that appropriate nutrients be provided to promote growth. Children need adequate caloric intake along with activity for gross-motor development. Dieting might not provide the intake necessary.
question
You are working in an adolescent health center when a 15-yearold patient shares with you that she thinks she is pregnant and is worried that she may now have a sexually transmitted infection (STI). Her pregnancy test is negative. What is your next priority of care? A) Contact her parents to alert them of her need for birth control. B) Refer her to a primary health care provider to obtain a prescription for birth control. C) Counsel her on safe sex practices. D) Ask her to have her partner come to the clinic for STI testing.
answer
C. Adolescent pregnancy and STIs are concerns that should be addressed by the nurse to support health care.
question
While working in the high-school clinic, one of the students tells you that she is worried about her friend who has started to refuse to participate in group activities, no longer cares about how she looks at school, and is not going to all of her classes. Your assessment of these symptoms may indicate that: A) She has just broken up with her boyfriend and time will heal all. B) You will need to observe her over time to see if symptoms persist. C) School may be too difficult for her right now. D) She may be at increased risk for suicide.
answer
D. Depression is a major health concern for adolescents and can be triggered by many factors; the symptoms that are listed indicate increased risk.
question
With the exception of pregnant or lactating women, the young adult has usually completed physical growth by the age of: A) 18. B) 20. C) 25. D) 30.
answer
B. Physical growth in the young adult is usually completed by the age of 20.
question
The nurse is completing an assessment on a male patient, age 24. Following the assessment, the nurse notes that his physical and laboratory findings are within normal limits. Because of these findings, nursing interventions are directed toward activities related to: A) Instructing him to return in 2 years. B) Instructing him in secondary prevention. C) Instructing him in health promotion activities. D) Implementing primary prevention with vaccines
answer
C. Although young adults generally have a minimum of major health problems, lifestyles such as tobacco or alcohol abuse, risky sexual activity, obesity, and lack of physical activity put them at risk for health problems. Instructing them in health-promotion activities can decrease lifestyle-related health issues.
question
When determining the amount of information that a patient needs to make decisions about the prescribed course of therapy, many factors affect the patient's compliance with the regimen, including educational level and socioeconomic factors. Which additional factor affects compliance? A) Gender B) Lifestyle C) Motivation D) Family history
answer
C. Motivation plays a key role in compliance with a prescribed course of therapy. However, motivation can be influenced by a variety of factors, including age, experience, family history, social support, and pressure from health care providers.
question
A patient is laboring with her first baby, which is coming 2 weeks early. Her husband is in the military and might not get back in time, and both families are unable to be with her during labor. The doctor decides to call in which of the following people employed by the birthing area to be a support person to be present during labor? A) Nurse B) Midwife C) Assistant D) Lay doula
answer
D. Although a nurse and midwife would be supportive during labor, the scope of practice for nurses and midwives far exceeds that of a support person present during labor. An assistant is incorrect. This woman has no family available to support her during the laboring process and delivery. A lay doula is a support person who is present during labor to assist women who have no other source of support.
question
A single young adult female interacts with a group of close friends from college and work. They celebrate birthdays and holidays together. In addition, they help one another through many stressors. She views these individuals as: A) Family. B) Siblings. C) Substitute parents. D) Alternative family structure.
answer
A. Families can be defined biologically, legally, or as a social network with personally constructed ties and ideologies
question
Sharing eating utensils with a person who has a contagious illness increases the risk of illness. This type of health risk arises from: A) Lifestyle. B) Community. C) Family history. D) Personal hygiene habits.
answer
D. In all age-groups personal hygiene habits can be risk factors for the spread of contagious diseases. Sharing eating utensils with a person who has a contagious illness increases the risk of illness.
question
A 50-year-old woman has elevated cholesterol profile values that increase her cardiovascular risk factor. One method to control this risk factor is to identify current diet trends and describe dietary changes to reduce the risk. This nursing activity is a form of: A) Referral. B) Counseling. C) Health education D) Stress management techniques.
answer
C. The nurse is engaged in health teaching as she offers dietary information to the woman to enable her to make decisions about her dietary health practices in an attempt to lower her cholesterol.
question
A 34-year-old female executive has a job with frequent deadlines. She notes that, when the deadlines appear, she has a tendency to eat high-fat, high-carbohydrate foods. She also explains that she gets frequent headaches and stomach pain during these deadlines. The nurse provides a number of options for the executive, and she chooses yoga. In this scenario yoga is used as a(n): A) Outpatient referral. B) Counseling technique. C) Health promotion activity. D) Stress-management technique.
answer
D. Relaxation techniques such as imagery, biofeedback, and yoga help recondition the patient's response to stress. Yoga is an ancient practice of controlling body and mind through physical and mental harmony. It is frequently used as an effective intervention for stress and stress-related physical symptoms.
question
A 50-year-old male patient is seen in the clinic. He tells the nurse that he has recently lost his job and his wife of 26 years has asked for a divorce. He has a flat affect. Family history reveals that his father committed suicide at the age of 53. The nurse should assess for the following: A) Cardiovascular disease B) Depression C) Sexually transmitted infection D) Iron deficiency anemia
answer
B. Depression is common among middle-age adults and has a variety of causes. Risk factors for depression include disappointments or losses at work, at school, or in family relationships and family history.
question
Middle-age adults frequently find themselves trying to balance responsibilities related to employment, family life, care of children, and care of aging parents. People finding themselves in this situation are frequently referred to as being a part of: A) The sandwich generation. B) The millennial generation. C) Generation X. D) Generation Y.
answer
A. The sandwich generation is a generation of people who care for their aging parents while supporting their own children. According to the Pew Research Center, just over one in every eight Americans ages 40 to 60 is both raising a child and caring for a parent.
question
Intimate partner violence (IPV) is linked to which of the following factors? (Select all that apply.) A) Alcohol abuse B) Pregnancy C) Unemployment D) Drug use
answer
A,B,C,D. IPV has been linked to alcohol abuse, especially heavy drinking; stress from unemployment of the perpetrator; drug use; and pregnancy. The greatest risk of IPV occurs during the reproductive years, with a pregnant woman having a 35.6% greater risk of being a victim of IPV than a nonpregnant woman.
question
Sexually transmitted infections (STIs) continue to be a major health problem in young adults. Men ages 20 to 24 years have the highest rate of which STI? A) Chlamydia B) Syphilis C) Gonorrhea D) Herpes zoster
answer
A. In 2008 20- to 24-year-old men had the highest rate of chlamydia among all men (1056.1 per 100,000 population), with chlamydia rates in men of this age-group increasing by 12.6% from the previous year.
question
Formation of positive health habits may prevent the development of chronic illness later in life. Which of the following are examples of positive health habits? (Select all that apply.) A) Routine screening and diagnostic tests B) Unprotected sexual activity C) Regular exercise D) Excess alcohol consumption
answer
A,C. Routine screening and diagnostic tests (e.g., laboratory screening for serum cholesterol or serum glucose levels, mammography, or colonoscopy) provide early detection of health issues. Regular exercise helps to maintain weight and improve musculoskeletal functioning.
question
Chronic illness (e.g., diabetes mellitus, hypertension, rheumatoid arthritis) may affect a person's roles and responsibilities during middle adulthood. When assessing the knowledge base of both the middle-age patient with a chronic illness and his family, the assessment should include which of the following? (Select all that apply.) A) The medical course of the illness B) The prognosis for the patient C) Coping mechanisms of the patient and family D) The need for community and social services
answer
A,B,C,D. When assessing the patient with a chronic illness, it is important that the nurse know how much the patient and his family know about how the illness has progressed and the long-term prognosis for the patient. This includes understanding the patient's and families' ability and readiness to accept the illness and the outlook for the patient. Understanding the coping mechanisms used by the patient and family helps the nurse to determine how to teach and counsel them about his or her treatment regimen and whether or not community or social services are needed and will be accepted to help the patient and family.
question
A 45-year-old obese woman tells the nurse that she wants to lose weight. After conducting a thorough assessment, the nurse concludes that which of the following may be contributing factors to the woman's obesity? (Select all that apply.) A) The woman works in an executive position that is very demanding. B) The woman works out at the corporate gym at 5 am two mornings per week C) The woman says that she has little time to prepare meals at home and eats out at least four nights a week. D) The woman says that she tries to eat "low cholesterol" foods to help lose weight.
answer
A,C. Demanding and stressful work environments can lead to frequent stress eating of nonnutritious foods. Frequently eating away from home and eating fast food have been identified as contributing factors to obesity.
question
A student nurse is caring for a 78-year-old patient with multiple sclerosis. The patient has had an indwelling Foley catheter in for 3 days. Eight hours ago the patient's temperature was 37.1° C (98.8° F). The student reports her recent assessment to the registered nurse (RN): the patient's temperature is 37.2° C (99° F); the Foley catheter is still in place, draining dark urine; and the patient is uncertain what time of day it is. From what the RN knows about presentation of symptoms in older adults, what should he recommend? A) Tell the student that temporary confusion is normal and simply requires reorientation B) Tell the student to increase the patient's fluid intake since the urine is concentrated C) Tell the student that her assessment findings are normal for an older adult D) Tell the student that he will notify the physician of the findings
answer
D. The patient may have subtle symptoms of a urinary tract infection, as evidenced by a slight increase in body temperature, development of confusion, and the dark-colored urine. Temporary confusion is not a normal condition in older adults. Increasing the fluid intake is acceptable but not a recommendation for the set of symptoms the patient presents. The presenting set of symptoms is not normal.
question
A patient's family member is considering having her mother placed in a nursing center. You have talked with the family before and know that this is a difficult decision. Which of the following criteria would you recommend in choosing a nursing center? (Select all that apply.) A) The center should be clean, and rooms should look like a hospital room. B) There should be adequate staffing on all shifts. C) Social activities should be available for all residents. D) Three meals should be served daily with a set menu and serving schedule. E) Family involvement in care planning and assisting with physical care is necessary.
answer
B,C,E. Adequate staffing, provision of social activities, and active family involvement are essential. Meals should be high quality with options for what to eat and when it is served. A nursing center should be clean, but it should look like a person's home.
question
A nurse has conducted an assessment of a new patient who has come to the medical clinic. The patient is 82 years old and has had osteoarthritis for 10 years and diabetes mellitus for 20 years. He is alert but becomes easily distracted during the nursing history. He recently moved to a new apartment, and his pet beagle died just 2 months ago. He is most likely experiencing: A) Dementia. B) Depression. C) Delirium. D) Disengagement.
answer
B. Factors that often lead to depression include presence of a chronic disease or a recent change or life event (such as loss). Patients are alert but easily distracted in conversation.
question
A major life event such as the death of a loved one, a move to a nursing home, or a cancer diagnosis could precipitate: A) Dementia. B) Delirium. C) Depression. D) Stroke.
answer
C. The onset of depression could be abrupt or gradual, but the usual cause is a major life-altering event in the life of the person experiencing the depression.
question
Sexuality is maintained throughout our lives. Which answer below best explains sexuality in an older adult? A) When the sexual partner passes away, the survivor no longer feels sexual. B) A decrease in an older adult's libido occurs. C) Any outward expression of sexuality suggests that the older adult is having a developmental problem. D) All older adults, whether healthy or frail, need to express sexual feelings.
answer
D. Sexuality is normal throughout the life span, and older adults need to be able to express their sexual feelings
question
Older adults experience a change in sexual activity. Which best explains this change? A) The need to touch and be touched is decreased. B) The sexual preferences of older adults are not as diverse. C) Physical changes usually do not affect sexual functioning. D) Frequency and opportunities for sexual activity may decline.
answer
D. As a result of loss of a loved one or a chronic illness in themselves or their partner, opportunities for sexual activity may decline
question
You see a 76-year-old woman in the outpatient clinic. Her chief complaint is vision. She states she has really noticed glare in the lights at home. Her vision is blurred; and she is unable to play cards with her friends, read, or do her needlework. You suspect that she may have: A) Presbyopia. B) Disengagement. C) Cataract(s). D) Depression.
answer
C. Cataracts normally result in blurred vision, sensitivity to glare, and gradual loss of vision. Presbyopia is a common eye condition resulting in a person having difficulty adjusting to near and far vision. The symptoms are not reflective of depression since her vision affects her ability to interact. She has not chosen to avoid her friends. Disengagement is a term referring to aging theory.
question
A nurse is caring for a patient preparing for discharge from the hospital the next day. The patient does not read and has a hearing loss. His family caregiver will be visiting before discharge. What can you do to facilitate the patient's understanding of his discharge instructions? (Select all that apply.) A) Speak loudly so the patient can hear you. B) Sit facing the patient so he is able to watch your lip movements and facial expressions. C) Present one idea or concept at a time. D) Send a written copy of the instructions home with him and tell him to have the family review them. E) Include the family caregiver in the teaching session
answer
B,C,E. Teaching and communication are more effective with older adults when you sit and face the patient and present one idea or concept at a time. This requires planning. Speaking loudly can distort sound. Speak in a normal tone. Sending instructions is helpful but will not directly facilitate the patient's own understanding. Sharing information with a caregiver provides someone to clarify instructions.
question
Taste buds atrophy and lose sensitivity, and appetite may decrease. As a result, the older adult is less able to discern: A) Spicy and bland foods. B) Salty, sour, and bitter tastes. C) Hot and cold food temperatures. D) Moist and dry food preparations.
answer
B. Often an older adult uses "heavy" spices because of his or her inability to taste the food.
question
Kyphosis, a change in the musculoskeletal system, leads to: A) Decreased bone density in the vertebrae and hips. B) Increased risk for pathological stress fractures in the hips. C) Changes in the configuration of the spine that affect the lungs and thorax. D) Calcification of the bony tissues of the long bones such as in the legs and arm
answer
C. This can also affect the ability of the patient to deep breath and cough effectively.
question
A 63-year-old patient is retiring from his job at an accounting firm where he was in a management role for the past 20 years. He has been with the same company for 42 years and was a dedicated employee. His wife is a homemaker. She raised their five children, babysits for her grandchildren as needed, and belongs to numerous church committees. What are your major concerns for this patient? (Select all that apply.) A) The loss of his work role B) The risk of social isolation C) A determination if the wife will need to start working D) How the wife expects household tasks to be divided in the home in retirement E) The age the patient chose to retire
answer
A,D. The psychosocial stresses of retirement are usually related to role changes with a spouse or within the family and to loss of the work role. Often there are new expectations of the retired person. This patient is not likely to become socially isolated because of the size of the family. Whether the wife will have to work is not a major concern at this time, nor is the age of the patient.
question
During a home health visit a nurse talks with a patient and his family caregiver about the patient's medications. The patient has hypertension and renal disease. Which of the following findings places him at risk for an adverse drug event? (Select all that apply.) A) Taking two medications for hypertension B) Taking a total of eight different medications during the day. C) Having one physician who reviews all medications D) Patient's health history E) Involvement of the caregiver in assisting with medication administration
answer
B,D. The patient is at risk for an adverse drug event (ADE) because of polypharmacy and his history of renal disease, which affects drug excretion. Taking two medications for hypertension is common. Having one physician review all medications and involving a family caregiver are desirable and are safety factors for preventing ADEs.
question
You are caring for an 80-year-old man who recently lost his wife. He shares with you that he has been drinking more than he ever did in the past and feels hopeless without his wife. He reports that he rarely sees his children and feels isolated and alone. This patient is at risk for: A) Dementia. B) Liver failure. C) Dehydration. D) Suicide.
answer
D. The patient is sharing that he is depressed. Key concepts include recent loss of his wife, excessive drinking, hopelessness, and isolation, making him at risk for suicide.
question
You are working with an older adult after an acute hospitalization. Your goal is to help this person be more in touch with time, place, and person. What might you try? A) Reminiscence B) Validation therapy C) Reality orientation D) Body image interventions
answer
C. Reality orientation is a communication technique that can help restore a sense of reality, improve level of awareness, promote socialization, elevate independent functioning, and minimize confusion.
question
A 71-year-old patient enters the emergency department after falling down stairs in the home. The nurse is conducting a fall history with the patient and his wife. They live in a one-level ranch home. He has had diabetes for over 15 years and experiences some numbness in his feet. He wears bifocal glasses. His blood pressure is stable around 130/70. The patient does not exercise regularly and complains of weakness in his legs when climbing stairs. He is alert, oriented, and able to answer questions clearly. What are the fall risk factors for this patient? (Select all that apply.) A) Presence of a chronic disease B) Impaired vision C) Residence design D) Blood pressure E) Leg weakness F) Exercise history
answer
B,E,F. Risk factors for falling include sensory changes such as visual loss, musculoskeletal conditions affecting mobility (in this case weakness), and deconditioning (from lack of exercise). The mere presence of a chronic disease is not a risk factor unless it is a condition such as a neurological disorder that alters mobility or cognitive function. The patient's blood pressure is stable, and there is no report of orthostatic hypotension. A one-floor residence should not pose risks.
question
A patient on bed rest for several days attempts to walk with assistance. He becomes dizzy and nauseated. His pulse rate jumps from 85 to 110 beats/min. These are most likely symptoms of which of the following? A) Rebound hypertension B) Orthostatic hypotension C) Dysfunctional proprioception. D) Central nervous system rebound hypotension
answer
B. Signs and symptoms of orthostatic hypotension include dizziness, light-headedness, nausea, tachycardia, pallor, and even fainting.
question
Which action(s) are appropriate for the nurse to implement when a patient experiences orthostatic hypotension? (Select all that apply.) A) Call for assistance. B) Allow patient to sit down. C) Take patient's blood pressure and pulse. D) Continue to ambulate patient to build endurance. E) If patient begins to faint, allow him to slide against the nurse's leg to the floor.
answer
A,B,C,E. If the patient has a fainting (syncope) episode or begins to fall, assume a wide base of support with one foot in front of the other, thus supporting the patient's body weight (see Fig. 38-5, A to C). Extend one leg and let the patient slide against it; gently lower the patient to the floor, protecting his or her head. Take the patient's blood pressure and pulse as soon as possible after incident.
question
Which of the following best motivates a patient to participate in an exercise program? A) Giving a patient information on exercise B) Providing information to the patient when the patient is ready to change behavior C) Explaining the importance of exercise when a patient is diagnosed with a chronic disease such as diabetes D) Following up with instructions after the health care provider tells a patient to begin an exercise program
answer
B. Patients are more open to developing an exercise program when they are at a stage of readiness to change their behavior. Once the patient is at the stage of readiness, collaborate with him or her to develop an exercise program that fits his or her needs and provide continued follow-up support and assistance until the exercise program becomes a daily routine.
question
Which of the following is a principle of proper body mechanics when lifting or carrying objects? A) Keep the knees in a locked position. B) Bend at the waist to maintain a center of gravity. C) Maintain a wide base of support. D) Hold objects away from the body for improved leverage.
answer
C. Maintaining a wide base of support allows for proper body mechanics. Locking the knees or bending at the waist causes strain on the lower back. Holding objects close to the body helps use the center of gravity for leverage.
question
Which group of patients is at most risk for severe injuries related to falls? A) Adolescents B) Older adults C) Toddlers D) Young children
answer
B. Some older adults walk more slowly and are less coordinated. They also take smaller steps, keeping their feet closer together, which decreases the base of support. Thus body balance is unstable, and they are at greater risk for falls and injuries
question
A nurse plans to provide education to the parents of school-aged children and includes which of the following result of children being less physically active outside of school? A) An increase in obesity B) An increase in heart disease C) Higher computer literacy D) Improved school attendance and grades
answer
A. It is increasingly clear that children are less active, resulting in an increase in childhood obesity. Strategies for physical activity incorporated early into a child's daily routine may provide a foundation for lifetime commitment to exercise and physical fitness.
question
A nursing assistive personnel asks for help to transfer a patient who is 125 pounds (56.8 kg) from the bed to a wheelchair. The patient is unable to assist. What is the nurse's best response? A) "As long as we use proper body mechanics, no one will get hurt." B) "The patient only weighs 125 lb. You don't need my assistance." C) "Call the lift-team for additional assistance." D) "The two of us can easily lift the patient."
answer
C. Body mechanics alone are not sufficient to prevent musculoskeletal injuries when positioning or transferring patients (see Table 38-1). Teaching the use of patient-handling equipment or the use of a lift-team in combination with proper body mechanics is more effective.
question
You are transferring a patient who weighs 320 lb (145.5 kg) from his bed to a chair. The patient has an order for partial weight bearing as a result of bilateral reconstructive knee surgery. Which of the following is the best technique for transfer? A) Use a transfer board. B) Obtain a stand assist device. C) Implement a three-person carry. D) Use the ceiling-mounted lift.
answer
D. The use of patient-handling equipment helps prevent injury to health care workers and patients.
question
Which is the correct gait when a patient is ascending stairs on crutches? A) A modified two-point gait. The affected leg is advanced between the crutches to the stairs. B) A modified three-point gait. The unaffected leg is advanced between the crutches to the stairs. C) A swing-through gait. D) A modified four-point gait. Both legs advance between the crutches to the stairs.
answer
B. When ascending stairs on crutches, the patient usually uses a modified three-point gait (see Fig. 38-13).
question
A patient recovering from bilateral knee replacements is prescribed bilateral partial weight bearing. You reinforce crutch walking knowing that which of the following crutch gaits is most appropriate for this patient? A) Two-point gait B) Three-point gait C) Four-point gait D) Swing-through gait
answer
A. The two-point gait requires at least partial weight bearing on each foot (see Fig. 38-12). The patient moves a crutch at the same time as the opposing leg, so that the crutch movements are similar to arm motion during normal walking.
question
A patient with a right knee replacement is prescribed no weight bearing on the right leg. You reinforce crutch walking knowing that which of the following crutch gaits is most appropriate for this patient? A) Two-point gait B) Three-point gait C) Four-point gait D) Swing-through gait
answer
B. Three-point alternating, or three-point, gait requires the patient to bear all of the weight on one foot. In a three-point gait, the patient bears weight on both crutches and then on the uninvolved leg, repeating the sequence (see Fig. 38-12, B).
question
A patient on week-long bed rest is now performing isometric exercises. Which nursing diagnosis best addresses the safety of this patient? A) Disturbed thought processes B) Impaired skin integrity C) Disturbed body image D) Risk for activity intolerance
answer
D. The nursing diagnosis, risk for activity intolerance, best relates to patient safety because of the potential for orthostatic hypotension associated with prolonged bed rest.
question
Which of the following activities does the nurse delegate to nursing assistive personnel in regard to crutch walking? (Select all that apply.) A) Notify nurse if patient reports pain before, during, or after exercise. B) Notify nurse of patient complaints of increased fatigue, dizziness, light-headedness when obtaining vital signs before and/or after exercise. C) Notify nurse of vital sign values. D) Evaluate the patient's ability to use crutches properly. E) Prepare the patient for exercise by assisting in dressing and putting on shoes.
answer
A,B,C,E. These are all correct as they are within the nursing assistive personnel activities (e.g., notifying the nurse or completing assigned activities). Evaluation is within the scope of professional nursing practice and is not delegated.
question
Select statements that apply to the proper use of a cane. (Select all that apply.) A) For maximum support when walking, the patient places the cane forward 15 to 25 cm (6 to 10 inches), keeping body weight on both legs. The weaker leg is moved forward to the cane so body weight is divided between the cane and the stronger leg. B) A person's cane length is equal to the distance between the elbow and the floor. C) Canes provide less support than a walker and are less stable. D) The patient needs to learn that two points of support such as both feet or one foot and the cane need to be present at all times.
answer
A,C,D. A person's cane length is equal to the distance between the greater trochanter and the floor. For maximum support when walking, the patient places the cane forward 15 to 25 cm (6 to 10 inches), keeping body weight on both legs. The patient needs to learn that two points of support (i.e., both feet or one foot and the cane) are present at all times.
question
A patient is discharged after an exacerbation of chronic obstructive pulmonary disease (COPD). She states, "I'm afraid to go to pulmonary rehabilitation." What is your best response? A) Pulmonary rehabilitation provides a safe environment for monitoring your progress. B) You have to participate or you will be back in the hospital. C) Tell me more about your concerns with going to pulmonary rehabilitation. D) The staff at our pulmonary rehabilitation facility are professionals and will not cause you any harm.
answer
A. Pulmonary rehabilitation is beneficial in helping patients reach an optimal level of functioning. Some patients are fearful of participating in exercise because of the potential of worsening dyspnea (difficulty breathing). Pulmonary rehabilitation provides a safe environment for monitoring the progress of patients.
question
An older adult has limited mobility as a result of a surgical repair of a fracture hip. During assessment you note that the patient cannot tolerate lying flat. Which of the following assessment data support a possible pulmonary problem related to impaired mobility? (Select all that apply.) A) B/P = 128/84 B) Respirations 26 per minute on room air C) HR 114 D) Crackles heard on auscultation E) Pain reported as 3 on scale of 0 to 10 after medication
answer
B,C,D. Patients with reduced mobility are at risk for retained pulmonary secretions, and this risk increases in postoperative patients. As a result of retained secretions, the respiratory rate increases. The heart rate also increases because the heart is trying to improve oxygen levels. These symptoms are of concern for older adults because, if left untreated, further complications such as heart failure can occur.
question
A patient has her call bell on and looks frightened when you enter the room. She has been on bed rest for 3 days following a fractured femur. She says, "It hurts when I try to breathe, and I can't catch my breath." Your first action is to: A) Call the health care provider to report this change in condition. B) Give the patient a paper bag to breathe into to decrease her anxiety. C) Assess her vital signs, perform a respiratory assessment, and be prepared to start oxygen. D) Explain that this is normal after such trauma and administer the ordered pain medication.
answer
C. These are signs of possible pulmonary emboli, which can be life threatening. You must assess your patient, be prepared to start oxygen, and have someone call the surgeon while you stay with the patient to continue to monitor her status.
question
The nurse puts elastic stockings on a patient following major abdominal surgery. The nurse teaches the patient that the stockings are used after a surgical procedure to: A) Prevent varicose veins. B) Prevent muscular atrophy. C) Ensure joint mobility and prevent contractures. D) Promote venous return to the heart.
answer
D. Elastic stockings maintain external pressure on the lower extremities and assist in promoting venous return to the heart. This increase in venous return helps reduce the stasis of blood and in turn reduces the risk for deep vein thrombosis (DVT) formation in the lower extremities.
question
A nurse is teaching a community group about ways to minimize the risk of developing osteoporosis. Which of the following statements made by a woman in the audience reflects a need for further education? A) "I usually go swimming with my family at the YMCA 3 times a week." B) "I need to ask my doctor if I should have a bone mineral density check this year." C) "If I don't drink milk at dinner, I'll eat broccoli or cabbage to get the calcium that I need in my diet." D) "I'll check the label of my multivitamin. If it has calcium, I can save money by not taking another pill. "
answer
D. Just because a multivitamin has calcium in it does not mean that the woman is receiving enough to meet her needs. She must know her requirement and make the decision based on that rather than on the value for calcium on the label.
question
The patient at greatest risk for developing multiple adverse effects of immobility is a: A) 1-year-old child with a hernia repair. B) 80-year-old woman who has suffered a hemorrhagic cerebrovascular accident (CVA). C) 51-year-old woman following a thyroidectomy. D) 38-year-old woman undergoing a hysterectomy.
answer
B. The older the patient and the greater the period of immobility, which can be significant following a hemorrhagic stroke, the greater is the number of systems that can be affected by the immobility.
question
An older adult who was in a car accident and fractured his femur has been immobilized for 5 days. Which nursing diagnosis is related to patient safety when the nurse assists this patient out of bed for the first time? A) Chronic pain B) Impaired skin integrity C) Risk for ineffective cerebral tissue perfusion D) Risk for activity intolerance
answer
D. Patients on bed rest are at risk for activity intolerance, which increases patients' risk for falling.
question
A patient had a left-sided cerebrovascular accident 3 days ago and is receiving 5000 units of heparin subcutaneously every 12 hours to prevent thrombophlebitis. The patient is receiving enteral feedings through a small-bore nasogastric (NG) tube because of dysphagia. Which of the following symptoms requires the nurse to call the health care provider immediately? A) Pale yellow urine B) Unilateral neglect C) Slight movement noted on the R side D) Coffee ground-like aspirate from the feeding tube
answer
D. When patients are receiving medications such as heparin or enoxaparin (Lovenox), you must assess for signs of bleeding. These include overt signs such as bleeding from their gums or covert signs, which can be detected by testing their stool or observing their aspirate from NG tubes for coffee ground-like matter. These are signs of bleeding in the gastrointestinal tract.
question
A home care nurse is preparing the home for a patient who is discharged to home following a left-sided stroke. The patient is cooperative and can ambulate with a quad-cane. Which of the following must be corrected or removed for the patient's safety? (Select all that apply.) A) The rubber mat in the walk-in shower B) The three-legged stool on wheels in the kitchen C) The braided throw rugs in the entry hallway and between the bedroom and bathroom D) The night-lights in the hallways, bedroom, and bathroom E) The cordless phone next to the patient's bed
answer
B,C. Stools on wheels and braided throw rugs are hazards that put the patient at risk for falls. By planning ahead and collaborating, the home care nurse can provide a safe home environment for the patient after discharge.
question
The nurse is caring for a patient whose calcium intake must increase because of high risk factors for osteoporosis. The nurse would recommend which of the following menus? A) Cream of broccoli soup with whole wheat crackers and tapioca for dessert B) Hamburger on soft roll with a side salad and an apple for dessert C) Low-fat turkey chili with sour cream and fresh pears for dessert D) Chicken salad on toast with tomato and lettuce and honey bun for dessert
answer
A. The dairy and broccoli in the soup, the whole grain crackers, plus the tapioca are all great sources of calcium.
question
Before transferring a patient from the bed to a stretcher, which assessment data does the nurse need to gather? (Select all that apply.) A) Patient's weight B) Patient's level of cooperation C) Patient's ability to assist D) Presence of medical equipment E) 24-hour calorie intake
answer
A,B,C,D. By assessing the patient thoroughly you make the correct decision concerning your ability to manage him or her safely, the need for additional personnel, the patient's ability or inability to assist you with the transfer, and the proper equipment to use for the transfer. The calorie intake for the past 24 hours does not affect safe transfer.
question
A patient of any age can develop a contracture of a joint when: A) The adductors muscles are weakened as a result of immobility. B) The muscle fibers become shortened because of disuse. C) The calcium-to-phosphorus ratio becomes disrupted. D) There is a deficiency in vitamin D.
answer
B. The adductor muscles are stronger than the abductor muscles; when patients are immobile and the joint is not exercised through their ROM, the adductor muscle fibers shorten, resulting in the contracture of that joint, which is usually permanent.
question
Immobilized patients are at risk for impaired skin integrity. Which of the following interventions would reduce this risk? (Select all that apply.) A) Repositioning patient every 1 to 2 hours while awake B) Using an objective, valid scale to assess patient's risk for pressure ulcer development C) Using a device to relieve pressure when patient is seated in chair D) Teaching patient how to shift weight at regular intervals while sitting in a chair E) A good rule is: the higher the risk for skin breakdown, the shorter the interval between position changes
answer
B,C,D,E. Patients must be repositioned around the clock, not just when they are awake. An objective assessment scale allows the nurse to assess for pressure ulcer risk over time. Once the risk is identified, the assessment tool guides the nurse in selecting appropriate pressure-relief devices. Showing the patient how to reduce his or her risk by shifting pressure is also important. Frequent and meaningful position changes that are in concert with the patient's condition and risk factors are necessary to reduce pressure ulcer developments.
question
Which of the following indicates that additional assistance is needed to transfer the patient from the bed to the stretcher? A) The patient is 5 feet 6 inches and weighs 120 lbs. B) The patient speaks and understands English. C) The patient received an injection of morphine 30 minutes ago for pain. D) You feel comfortable handling a patient of his size and with his level of cooperation
answer
C. The morphine injection would change the patient's ability to safely follow directions and participate in the transfer; therefore additional help would be needed to safely transfer the patient from the bed to the stretcher.
question
A patient with left-sided weakness asks his nurse, "Why are you walking on my left side? I can hold on to you better with my right hand." What would be your best therapeutic response? A) "Walking on your left side lets me use my right hand to hold on to your arm. In case you start to fall, I can still hold you." B) "Would you like me to walk on your right side so you feel more secure?" C) "Either side is appropriate, but I prefer the left side. If you like, I can have another nurse walk with you who will hold you on the right side." D) "By walking on your left side I can support you and help keep you from injury if you should start to fall. By holding your waist I would protect your shoulder if you should start to fall or faint.
answer
D. Walking on the affected (weak side) side and holding the patient around the waist or using a gait belt gives you better control if the patient starts to fall. If you were holding the patient's arm as he was falling, you might dislocate his shoulder.
question
Which is an outcome for a patient diagnosed with osteoporosis? A) Maintain serum level of calcium. B) Maintain independence with activities of daily living (ADLs). C) Reduce supplemental sources of vitamin D. D) Reverse bone loss through dietary manipulation.
answer
B. The main goal is to maintain independence in ADLs once osteoporosis is diagnosed. It is best to identify individuals at risk and work toward preventing the disease.
question
The nurse is assisting a patient with rheumatoid arthritis to bathe at the sink. During the bath the patient states that she is tired. The nurse notices the patient is breathing rapidly and the pulse is rapid. What is the nurse's best response? A) Finish the bath quickly B) Help the patient return to bed C) Leave the patient alone to rest in the chair at the sink for a few minutes D) Instruct the patient to take deep breaths and try to relax
answer
B. The report of fatigue and rapid respirations and pulse indicates that the patient is not tolerating the activity and needs to rest. Leaving the patient alone at the sink is not safe.
question
A patient who is cognitively impaired and has dementia requires hygiene care. The patient often displays aggressive behavior such as screaming and hitting during the bath. Which techniques make the bathing experience less stressful for both the nurse and the patient? (Select all that apply.) A) Allow the patient to perform as much of the care as possible. B) Start by washing the face. C) Try an alternative to traditional bathing such as the "bag bath." D) Use restraints to prevent the patient from injuring self or the nurse.
answer
A,C. Patients with cognitive impairment may respond to bathing by acting out aggressively. Studies have indicated that there are a number of triggering events, including washing the face first. The bag bath has been shown to result in a lower incidence of aggressive behavior than traditional bathing. Use of restraints is not warranted and can actually lead to injury as the patient often fights against the restraints.
question
What is the priority concern when providing oral hygiene for a patient who is unconscious? A) Thoroughly brushing all tooth and oral surfaces B) Preventing aspiration C) Controlling mouth odor D) Applying local antiseptic such as chlorhexidine
answer
B. Although thorough and effective cleaning is needed, measures to prevent aspiration of oral secretions and/or cleaning agents into the lungs take priority since aspiration can lead to lower respiratory infections.
question
A male nurse is caring for a 32-year-old female Muslim patient who has an indwelling Foley catheter. After introducing himself to the patient, the nurse learns that the patient does not want him to help her with personal hygiene care. Which of the following is(are) appropriate actions? (Select all that apply.) A) Finding a female nurse to help the patient B) Convincing the patient that he will work quickly and provide as much privacy as possible C) Skipping hygiene care for the day except for the parts that the patient can complete independently D) Asking the patient if she prefers a family member assist with the care
answer
A,D. Cultural variations affecting hygiene care include gender-congruent concerns. Pressuring patients to accept cultural values that they do not believe in and value is inappropriate. Patients with Foley catheters require routine perineal care; skipping care is not a safe practice.
question
You are helping a female patient bathe. As you are about to perform perineal care, the patient says, "I can finish my bath." The patient has discomfort and burning in the perineal area. What action do you need to take initially? A) Explain to the patient that, because of her symptoms, you need to observe the perineal area. B) Insist that you are supposed to complete the care. C) Honor the patient's request to complete her own perineal care to avoid any embarrassment. D) Ask the patient if a family member can complete the care instead.
answer
A. The symptoms of burning and discomfort indicate a problem. It is your responsibility to perform an assessment to note any vaginal or urethral discharge, skin irritation, and odors. It is not safe to let embarrassment cause you to overlook hygiene needs and the diagnosis of problems. Providing information and patient teaching often encourages patient cooperation.
question
Your patient wears full dentures. His usual denture care includes taking the teeth out once a day to brush. He wears the dentures overnight. You are concerned that he might be at risk for developing denture-induced stomatitis. Which points do you include in a teaching plan for denture care? (Select all that apply.) A) Remove dentures overnight once a week while they soak in a cleansing bath. B) Do not wear damaged or poorly fitting dentures. C) Observe mouth for reddened areas under the dentures and small red sores on the roof of the mouth. D) See dentist regularly. E) Rinse dentures after meals. F) Clean dentures every night with cleanser, rinsing well before replacing in mouth at bedtime.
answer
B,C,D,E. Denture stomatitis is caused by wearing dentures that do not fit well, wearing dentures overnight, and poor denture-cleaning habits that promote buildup of the yeast Candida albicans. The development of denture stomatitis is prevented by interfering with the buildup by removing the dentures overnight; cleaning properly; and preventing damage to the oral mucosa, gums, and palate.
question
A patient who is receiving chemotherapy has inflamed gums and oral mucosa and painful sores in the mouth. Which of the following oral care actions are appropriate? (Select all that apply.) A) Decreasing frequency of oral hygiene B) Applying water-soluble moisturizing gel on the oral mucosa C) Encouraging intake of soft foods D) Using commercial mouthwash
answer
B,C. Many commercial mouthwashes contain alcohol, which dries oral mucosa and causes pain. Although the mouth is tender, more frequent-than-usual care is needed to help hydrate tissue and prevent infection. Preventing drying out of the oral tissue helps healing. Eating soft foods is more comfortable for the patient.
question
While planning morning care, which of the following patients would receive the highest priority to receive his or her bath first? A) A patient who just returned to the nursing unit from surgery and is experiencing pain at a level of 7 on a scale of 0 to 10 B) A patient who prefers a bath in the evening when his wife visits and can help him C) A patient who is experiencing frequent incontinent diarrheal stools D) A patient who has just returned from diagnostic testing and complains of being very fatigued
answer
C. Patients who have body fluids, excretions, secretions, or wastes on the skin require immediate hygiene care. Hygiene care can be delayed for patients with pain or fatigue until these symptoms are controlled as long as there is no compelling reason such as drainage. Allowing patients to make choices involves them in care and gives them control.
question
During bathing your patient experiences shortness of breath and labored breathing with a respiratory rate of 30. The bed is in a flat position. You change the bed position to: A) Trendelenburg's. B) Reverse Trendelenburg's. C) Fowler's. D) Semi-Fowler's.
answer
C. Fowler's upright sitting position facilitates breathing by allowing for full expansion of the chest and lungs. Although reverse Trendelenburg's position raises the head of the bed, it is a straight tilt position and is not likely as comfortable as the more supported Fowler's position.
question
A nurse caring for a male patient observes the nursing assistive personnel (NAP) performing perineal care. Which of the following observed actions indicates a need for further teaching for the NAP? The NAP: A) Used clean gloves. B) Did not retract the foreskin before cleansing. C) Used the clean portion of washcloth for each cleansing wipe. D) Used a circular motion to cleanse from urinary meatus outward.
answer
B. Secretions collect beneath the foreskin and can promote bacterial growth if not removed.
question
A nurse teaching a family member caregiver how to bathe the patient explains the importance of using long strokes on the patient's extremities, moving from distal to proximal. Which explanation does the nurse include? Long strokes moving from distal to proximal are used to: A) Decrease the chance of infection. B) Help remove dry, flaky skin. C) Prevent skin trauma. D) Stimulate venous return.
answer
D. The pressure from long, smooth strokes moving from distal to proximal areas presses on the veins, which promotes venous return.
question
Which of the following actions would best help prevent skin breakdown in a patient who is incontinent of stools and very weak and drowsy? A) Checking frequently for soiling B) Washing the perineal area with strong soap and water C) Placing the call light within easy reach D) Keeping a pad under the patient
answer
A. Loose stool contains digestive enzymes that irritate the skin and need to be cleaned from the skin as soon as possible after soiling to prevent skin breakdown.
question
The nurse is caring for a patient who has reduced sensation in both feet. Which of the following should the nurse do? (Select all that apply.) A) Avoid cleaning the feet until an order from the health care provider is received. B) Wash the feet with lukewarm water and then dry well. C) Apply moisturizing lotion to the feet, especially between the toes. D) File the toenails straight across.
answer
B,D. Lukewarm water limits potential for burns in the patient with reduced sensation. Drying limits moisture that promotes growth of microorganisms. Always clip and file nails straight across (when ordered). An order is not needed to clean feet. Lotion between toes could cause maceration.
question
The nurse recognizes that her older-adult patient needs additional teaching about skin care when the older adult says, "I should: A) Bathe twice a week. B) Rinse well after using soap. C) Use hot water for bathing. D) Drink plenty of fluids.
answer
C. Hot water dries the skin by removing natural oils more quickly.
question
You ask the nursing assistive personnel (NAP) to clean a patient who has been incontinent of urine. Several minutes later you pass the open door of the room and see the NAP changing the patient's gown and linen. Which of the following requires your immediate attention? A) Room temperature is overly warm. B) Room door is open to the hallway. C) Television volume is too loud. D) Strong odor of urine is detected.
answer
B. This violates the patient's privacy. Although attention to the room temperature, noise level, and odor is required, the immediate concern is with privacy.
question
Which statement made by an adult patient demonstrates understanding of healthy nutrition teaching? A) I need to stop eating red meat. B) I will increase the servings of fruit juice to four a day. C) I will make sure that I eat a balanced diet and exercise regularly. D) I will not eat so many dark green vegetables and eat more yellow vegetables.
answer
C. Obesity is an epidemic in the United States. Proposed contributing factors are sedentary lifestyle and poor meal choices. Healthy eating and participation in exercise or other activities of healthy living promote good health.
question
The nurse teaches a patient who has had surgery to increase which nutrient to help with tissue repair? A) Fat B) Protein C) Vitamin D) Carbohydrate
answer
B. Proteins provide a source of energy (4 kcal/g), and they are essential for synthesis (building) of body tissue in growth, maintenance, and repair. Collagen, hormones, enzymes, immune cells, deoxyribonucleic acid (DNA), and ribonucleic acid (RNA) are all made of protein.
question
The nurse is caring for a patient experiencing dysphagia. Which interventions help decrease the risk of aspiration during feeding? (Select all that apply.) A) Sit the patient upright in a chair. B) Give liquids at the end of the meal. C) Place food in the strong side of the mouth. D) Provide thin foods to make it easier to swallow. E) Feed the patient slowly, allowing time to chew and swallow. F) Encourage patient to lie down to rest for 30 minutes after eating.
answer
A,C,E. Patients with dysphagia are at risk for aspiration and need more assistance with feeding and swallowing. Feed the patient with dysphagia slowly, providing smaller-size bites, and allow the patient to chew thoroughly and swallow the bite before taking another. Position the patient in an upright, seated position in a chair or raise the head of the bed to 90 degrees. If the patient has unilateral weakness, teach him or her and caregiver to place food in the stronger side of the mouth. Additional interventions include providing a 30-minute rest period before eating. Have the patient slightly flex the head to a chin-down position to help prevent aspiration. Determine the viscosity of foods that the patient tolerates best through the use of trials of different consistencies of foods and fluids. Thicker fluids are generally easier to swallow. More frequent chewing and swallowing assessments throughout the meal are necessary. Allow the patient time to empty the mouth after each spoonful, matching the speed of feeding to the patient's readiness. If the patient begins to cough or choke, remove the food immediately.
question
The nurse suspects that the patient receiving parenteral nutrition (PN) through a central venous catheter (CVC) has an air embolus. What action does the nurse need to take first? A) Raise head of bed to 90 degrees B) Turn patient to left lateral decubitus position C) Notify health care provider immediately D) Have patient perform the Valsalva maneuver
answer
B. An air embolus possibly occurs during insertion of the catheter or when changing the tubing or cap. Have the patient assume a left lateral decubitus position first. Then have the patient perform a Valsalva maneuver (holding the breath and "bearing down"). The increased venous pressure created by the maneuver prevents air from entering the bloodstream during catheter insertion. Maintaining integrity of the closed intravenous system also helps prevent air embolus.
question
Which action is initially taken by the nurse to verify correct position of a newly placed small-bore feeding tube? A) Placing an order for x-ray film examination to check position B) Confirming the distal mark on the feeding tube after taping C) Testing the pH of the gastric contents and observing the color D) Auscultating over the gastric area as air is injected into the tube
answer
A. At present the most reliable method for verification of placement of small-bore feeding tubes is x-ray film examination. The measurement of the pH of gastric secretions withdrawn from the feeding tube helps to determine the location of the tube. Auscultation has repeatedly been shown to be ineffective in detecting tubes accidentally placed in the lung. Further, it is not effective in distinguishing between gastric and intestinal placement for feeding tubes.
question
Based on knowledge of peptic ulcer disease (PUD), the nurse anticipates the presence of which bacteria when reviewing the laboratory data for a patient suspected of having PUD? A) Micrococcus B) Staphylococcus C) Corynebacterium D) Helicobacter pylori
answer
D. Marshall and Warren first identified Helicobacter pylori in 1984. It is a bacteria that causes up to 85% of peptic ulcers and is confirmed by laboratory tests. It is treated with antibiotics that control the bacterial infection.
question
The nurse is assessing a patient receiving enteral feedings via a small-bore nasogastric tube. Which assessment findings need further intervention? A) Gastric pH of 4.0 during placement check B) Weight gain of 1 pound over the course of a week C) Active bowel sounds in the four abdominal quadrants D) Gastric residual aspirate of 350 mL for the second consecutive time
answer
D. Delayed gastric emptying is a concern if 250 mL or more remains in the patient's stomach on each of two consecutive assessments. The North American Summit on Aspiration in the Critically Ill Patient made the following recommendations regarding gastric residual volumes (GRVs): (1) stop feedings immediately if aspiration occurs; (2) withhold feedings and reassess patient tolerance to feedings if GRV is over 500 mL for two successive measurements; and (3) routinely evaluate the patient for aspiration and use nursing measures to reduce the risk of aspiration if GRV is between 250 and 500 mL.
question
The home care nurse is seeing the following patients. Which patient is at greatest risk for experiencing inadequate nutrition? A) A 55-year-old obese man recently diagnosed with diabetes mellitus B) A recently widowed 76-year-old woman recovering from a mild stroke C) A 22-year-old mother with a 3-year-old toddler who had tonsillectomy surgery D) A 46-year-old man recovering at home following coronary artery bypass surgery
answer
B. Older adults who are homebound and have a chronic illness have additional nutritional risks. Frequently this group lives alone with few or no social or financial resources to assist in obtaining or preparing nutritionally sound meals. This contributes to a risk for food insecurity caused by low income and poverty. In addition, the mild stroke might cause dysphagia.
question
Which statement made by a patient of a 2-month-old infant requires further education? A) I'll continue to use formula for the baby until he is a least a year old. B) I'll make sure that I purchase iron-fortified formula. C) I'll start feeding the baby cereal at 4 months. D) I'm going to alternate formula with whole milk starting next month
answer
D. Infants should not have regular cow's milk during the first year of life. It causes gastrointestinal bleeding, is too concentrated for the infant's kidneys to manage, increases the risk of milk product allergies, and is a poor source of iron and vitamins C and E. Breast milk or formula provides sufficient nutrition for the first 4 to 6 months of life. The development of fine-motor skills of the hand and fingers parallels the infant's interest in food and self-feeding. Iron-fortified cereals are typically the first semisolid food to be introduced. For infants 4 to 11 months, cereals are the most important nonmilk source of protein.
question
The nurse is teaching a program on healthy nutrition at the senior community center. Which points should be included in the program for older adults? (Select all that apply.) A) Avoid grapefruit and grapefruit juice, which impair drug absorption. B) Increase the amount of carbohydrates for energy. C) Take a multivitamin that includes vitamin D for bone health. D) Cheese and eggs are good sources of protein. E) Limit fluids to decrease the risk of edema.
answer
A,C,D. Caution older adults to avoid grapefruit and grapefruit juice because these impair absorption of many drugs. Thirst sensation diminishes, leading to inadequate fluid intake or dehydration; thus older adults should be encouraged to ingest adequate fluids. Some older adults avoid meats because of cost or because they are difficult to chew. Cream soups and meat-based vegetable soups are nutrient-dense sources of protein. Cheese, eggs, and peanut butter are also useful high-protein alternatives. Milk continues to be an important food for older women and men who need adequate calcium to protect against osteoporosis (a decrease of bone mass density). Screening and treatment are necessary for both older men and women. Vitamin D supplements are important for improving strength and balance, strengthening bone health, and preventing bone fractures.
question
The nurse sees the nursing assistive personnel (NAP) perform the following for a patient receiving continuous enteral feedings. What intervention does the nurse need to address immediately with the NAP? The NAP: A) Fastens the tube to the gown with tape. B) Places the patient supine while giving a bath. C) Performs oral care for the patient. D) Elevates the head of the bed 45 degrees.
answer
B. Patients receiving enteral feedings should have the head of the bed elevated a minimum of 30 degrees, preferably 45 degrees, unless medically contraindicated. Laying the patient supine increases the risk of aspiration of the feeding and should be avoided. This needs to be addressed to maintain patient safety.
question
The patient receiving total parenteral nutrition (TPN) asks the nurse why his blood glucose is being checked since he does not have diabetes. What is the best response by the nurse? A) TPN can cause hyperglycemia, and it is important to keep your blood glucose level in an acceptable range. B) The high concentration of dextrose in the TPN can give you diabetes; thus you need to be monitored closely. C) Monitoring your blood glucose level helps to determine the dose of insulin that you need to absorb the TPN. D) Checking your blood glucose level regularly helps to determine if the TPN is effective as a nutrition intervention.
answer
A. The TPN formula is a combination of crystalline amino acids, hypertonic dextrose, electrolytes, vitamins, and trace elements. Administration of concentrated glucose is accompanied by increases in endogenous insulin production, which causes cations (potassium, magnesium, and phosphorus) to move intracellularly. Blood glucose levels should be monitored every 6 hours to assess for hyperglycemia. Maintaining blood glucose within acceptable limits helps prevent complications from the TPN.
question
A patient needs to learn to use a walker. Which domain is required for learning this skill? A) Affective domain B) Cognitive domain C) Attentional domain D) Psychomotor domain
answer
D. Using a walker requires the integration of mental and muscular activity.
question
The nurse is planning to teach a patient about the importance of exercise. When is the best time for teaching to occur? (Select all that apply.) A) When there are visitors in the room B) When the patient's pain medications are working C) Just before lunch, when the patient is most awake and alert D) When the patient is talking about current stressors in his or her life
answer
B,C. Plan teaching when the patient is most attentive, receptive, alert, and comfortable.
question
A patient newly diagnosed with cervical cancer is going home. The patient is avoiding discussion of her illness and postoperative orders. What is the nurse's best plan in teaching this patient? A) Teach the patient's spouse B) Focus on knowledge the patient will need in a few weeks C) Provide only the information that the patient needs to go home D) Convince the patient that learning about her health is necessary
answer
C. This patient is in denial; thus it is appropriate to only give her information that is needed immediately.
question
The school nurse is about to teach a freshman-level high school health class about nutrition. What is the best instructional approach to ensure that the students meet the learning outcomes? A) Provide information using a lecture B) Use simple words to promote understanding C) Develop topics for discussion that require problem solving D) Complete an extensive literature search focusing on eating disorders
answer
C. Adolescents learn best when they are able to use problem solving to help them make choices.
question
A nurse is going to teach a patient how to perform breast self-examination. Which behavioral objective does the nurse set to best measure the patient's ability to perform the examination? A) The patient will verbalize the steps involved in breast self-examination within 1 week. B) The nurse will explain the importance of performing breast self-examination once a month. C) The patient will perform breast self-examination correctly on herself before the end of the teaching session. D) The nurse will demonstrate breast self-examination on a breast model provided by the American Cancer Society.
answer
C. Return demonstration provides an excellent source of feedback and reinforcement to evaluate learning.
question
A patient with chest pain is having an emergency cardiac catheterization. Which teaching approach does the nurse use in this situation? A) Telling approach B) Selling approach C) Entrusting approach D) Participating approach
answer
A. The telling approach is most appropriate when preparing a patient for an emergency procedure.
question
The nurse is teaching a parenting class to a group of pregnant adolescents. The nurse pretends to be the baby's father, and the adolescent mother is asked to show how she would respond to the father if he gave her a can of beer. Which teaching approach did the nurse use? A) Role play B) Discovery C) An analogy D) A demonstration
answer
A. In role play people are asked to play themselves or someone else in a situation to enhance their confidence in handling that situation in the future.
question
An older adult is being started on a new antihypertensive medication. In teaching the patient about the medication, the nurse: A) Speaks loudly. B) Presents the information once. C) Expects the patient to understand the information quickly. D) Allows the patient time to express himself or herself and ask questions.
answer
D. When teaching older adults, it is important to establish rapport, involve them in their care, and allow them to progress at their own pace
question
A patient needs to learn how to administer a subcutaneous injection. Which of the following reflects that the patient is ready to learn? A) Describing difficulties a family member has had in taking insulin B) Expressing the importance of learning the skill correctly C) Being able to see and understand the markings on the syringe D) Having the dexterity needed to prepare and inject the medication
answer
B. Patients are ready to learn when they understand the importance of learning and are motivated to learn.
question
A patient who is hospitalized has just been diagnosed with diabetes. He is going to need to learn how to give himself injections. Which teaching method does the nurse use? A) Simulation B) Demonstration C) Group instruction D) One-on-one discussion
answer
B. Demonstration is used to help patients learn psychomotor skills.
question
When a nurse is teaching a patient about how to administer an epinephrine injection in case of a severe allergic reaction, he or she tells the patient to hold the injection like a dart. Which of the following instructional methods did the nurse use? A) Telling B) Analogy C) Demonstration D) Simulation
answer
B. Analogies use familiar images when teaching to help explain complex information.
question
A nurse needs to teach a young woman newly diagnosed with asthma how to manage her disease. Which of the following topics does the nurse teach first? A) How to use an inhaler during an asthma attack B) The need to avoid people who smoke to prevent asthma attacks C) Where to purchase a medical alert bracelet that says she has asthma D) The importance of maintaining a healthy diet and exercising regularly
answer
A. It is important to start with essential life-saving information when teaching people because they usually remember what you tell them first.
question
A nurse is teaching a group of young college-age women the importance of using sunscreen when going out in the sun. What type of content is the nurse providing? A) Simulation B) Restoring health C) Coping with impaired function D) Health promotion and illness prevention
answer
D. Health promotion and illness prevention are the focus when nurses provide information to help patients improve their health and avoid illness.
question
A nurse is planning a teaching session about healthy nutrition with a group of children who are in first grade. The nurse determines that after the teaching session the children will be able to name three examples of foods that are fruits. This is an example of: A) A teaching plan. B) A learning objective. C) Reinforcement of content. D) Enhancing the children's self-efficacy.
answer
B. A learning objective describes what the learner will do after the teaching session.
question
A nurse is teaching a 27-year-old gentleman how to adjust his insulin dosages based on his blood sugar results. What type of learning is this? A) Cognitive B) Affective C) Adaptation D) Psychomotor
answer
A. Cognitive learning requires thinking; learning how to adjust insulin requires analysis, synthesis, and evaluation, which are all types of cognitive learning.
question
A manager who is reviewing the nurses' notes in a patient's medical record finds the following entry, "Patient is difficult to care for, refuses suggestion for improving appetite." Which of the following directions does the manager give to the staff nurse who entered the note? A) Avoid rushing when charting an entry. B) Use correction fluid to remove the entry. C) Draw a single line through the statement and initial it. D) Enter only objective and factual information about the patient.
answer
D. Nurses should enter only objective and factual information about patients. Opinions have no place in the medical record. Because the information has already been entered and is not incorrect, it should be left on the record. Never use correction fluid in a written medical record.
question
A new graduate nurse is providing a telephone report to a patient's health care provider and accepting telephone orders from the provider. Which of the following actions requires the new nurse's preceptor to intervene? The new nurse: A) Uses SBAR (Situation-Background-Assessment-Recommendation) as a format when providing the report. B) Gives a newly ordered medication before entering the order in the patient's medical record. C) Reads the orders back to the health care provider after receiving them and verifies their accuracy. D) Asks the preceptor to listen in on the phone conversation.
answer
B. Nurses enter orders into the computer or write them on the order sheet as they are being given to allow the read-back process to occur.
question
As you enter the patient's room, you notice that he is anxious to say something. He quickly states, "I don't know what's going on; I can't get an explanation from my doctor about my test results. I want something done about this." Which of the following is the most appropriate documentation of the patient's emotional status? A) The patient has a defiant attitude and is demanding his test results. B) The patient appears to be upset with his nurse because he wants his test results immediately. C) The patient is demanding and complains frequently about his doctor. D) The patient stated that he felt frustrated by the lack of information he received regarding his tests
answer
D. This is a nonjudgmental statement regarding the nurse's observations about the patient. Documenting that the patient had a defiant attitude or was demanding and frequently complaining is judgmental, and information in the medical record should be factual and nonjudgmental. Documenting that the patient appears upset needs to be more specific regarding the reason for the patient's concern.
question
You are reviewing Health Insurance Portability and Accountability Act (HIPAA) regulations with your patient during the admission process. The patient states, "I've heard a lot about these HIPAA regulations in the news lately. How will they affect my care?" Which of the following is the best response? A) HIPAA allows all hospital staff access to your medical record. B) HIPAA limits the information that is documented in your medical record. C) HIPAA provides you with greater control over your personal health care information. D) HIPAA enables health care institutions to release all of your personal information to improve continuity of care.
answer
C. HIPAA provides patients with control over who receives and accesses their medical records. It does not allow uncontrolled access to the medical records. HIPAA also does not dictate what must be documented in the patient's medical record.
question
A patient asks for a copy of her medical record. The best response by the nurse is to: A) State that only her family may read the record. B) Indicate that she has the right to read her record. C) Tell her that she is not allowed to read her record. D) Explain that only health care workers have access to her record.
answer
B.Patients have the right to read their medical records, but the nurse should always know the facility policy regarding personal access to medical records because some require a nurse manager or other official to be present to answer questions about what is in the record. Families may read the records only when the patient has given permission.
question
Which of the following charting entries is most accurate? A) Patient walked up and down hallway with assistance, tolerated well. B) Patient up, out of bed, walked down hallway and back to room, tolerated well. C) Patient up, walked 50 feet and back down hallway with assistance from nurse. Spouse also accompanied patient during the walk. D) Patient walked 50 feet and back down hallway with assistance from nurse; HR 88 and regular before exercise, 94 and regular following exercise.
answer
D. The statement "Patient walked 50 feet and back down hallway with assistance from nurse; HR 88 and regular before exercise, 94 and regular following exercise" provides the most accurate, objective information for the chart.
question
Match the correct entry with the appropriate SOAP (Subjective—Objective—Assessment—Plan) category. A. Repositioned patient on right side. Encouraged patient to use patient-controlled analgesia (PCA) device. B. "The pain increases every time I try to turn on my left side." C. Acute pain related to tissue injury from surgical incision. D. Left lower abdominal surgical incision, 3 inches in length, closed, sutures intact, no drainage. Pain noted on mild palpation. S O A P
answer
"The pain increases every time I try to turn on my left side.":S, Left lower abdominal surgical incision, 3 inches in length, closed, sutures intact, no drainage. Pain noted on mild palpation.:O, Acute pain related to tissue injury from surgical incision.:A, Repositioned patient on right side. Encouraged patient to use patient-controlled analgesia (PCA) device.:P
question
On the nursing unit you are able to access a patient's medical record and review the education that other nurses provided to the patient during an initial hospitalization and three subsequent clinic visits. This type of feature is most common in what type of record system? A) Information technology. B) Electronic health record. C) Personal health information. D) Administrative information system.
answer
B. This is an example of an electronic health record. The electronic health record is an electronic record of patient health information generated whenever a patient accesses medical care in any health care delivery setting. In this question you are able to access information about the patient from the current hospitalization and from four previous times when the patient accessed care.
question
You are giving a hand-off report to another nurse who will be caring for your patient at the end of your shift. Which of the following pieces of information do you include in the report? (Select all that apply.) A) The patient's name, age, and admitting diagnosis B) Allergies to food and medications C) Your evaluation that the patient is "needy" D) How much the patient ate for breakfast E) That the patient's pain rating went from 8 to 2 on a scale of 1 to 10 after receiving 650 mg of Tylenol
answer
A,B,E. During change of shift report, include essential background information such as the patient's name, age, diagnosis, and allergies. Also include response to treatments such as response to pain-relieving measures. Information about how much the patient ate for breakfast is not necessary. This information is in the chart if the nurse really needs to know. Do not include critical comments about your patients.
question
You are supervising a beginning nursing student who is documenting patient care. Which of the following actions requires you to intervene? The nursing student: A) Documented medication given by another nursing student. B) Included the date and time of all entries in the chart. C) Stood with his back against the wall while documenting on the computer. D) Signed all documentation electronically.
answer
A. Nurses only document the care they provide; entries in the chart need to be dated, timed, and signed.
question
A group of nurses is discussing the advantages of using computerized provider order entry (CPOE). Which of the following statements indicates that the nurses understand the major advantage of using CPOE? A) "CPOE reduces transcription errors." B) "CPOE reduces the time necessary for health care providers to write orders." C) "Health care providers can write orders from any computer that has Internet access." D) "CPOE reduces the time nurses use to communicate with health care providers."
answer
A. CPOE eliminates the need for someone to transcribe the orders because it allows the provider to enter the order directly.
question
You are helping to design a new patient discharge teaching sheet that will go home with patients who are discharged to home from your unit. Which of the following do you need to remember when designing the teaching sheet? A) The new federal laws require that teaching sheets be e-mailed to patients after they are discharged. B) You need to use words the patients can understand when writing the directions. C) The form needs to be given to patients in a sealed envelope to protect their health information. D) The names of everyone who cared for the patient in the hospital need to be included on the form in case the patient has questions at home.
answer
B. Patients need to be able to understand information that you provide to them; ensure that written instructions are provided at a level that matches the patients' reading ability.
question
A nurse caring for a patient on a ventilator electronically documents the head of bed elevated at 20 degrees. Suddenly an alert warning appears on the screen warning the nurse that this patient is at a high risk for aspiration because the head of the bed is not elevated high enough. This warning is known as what type of system? A) Electronic health record B) Clinical documentation C) Clinical decision support system D) Computerized physician order entry
answer
C. A clinical decision support system is based on rules that are triggered by data entry. When certain rules are not met, alerts, warnings, or other information may be provided to the user.
question
While reviewing the pulmonary section of a patient's electronic chart, the physician notices blank spaces since the initial assessment the previous day when the nurse documented that the lung assessment was within normal limits. There also are no progress notes about the patient's respiratory status in the nurse's notes. The most likely reason for this is because: A) The nurses forgot to document on the pulmonary system. B) The nurses were charting by exception. C) The computer is not working correctly. D) The physician does not have authorization to view the nursing assessment.
answer
B. Given that the initial assessment indicated that the pulmonary system was within normal limits, the facility is most likely documenting by exception. There is no need for further documentation unless the pulmonary assessment changes and is no longer within normal limits.
question
What is an appropriate way for a nurse to dispose of printed patient information? A) Rip several times and place in a standard trash can B) Place in the patient's paper-based chart C) Place in a secure canister marked for shredding D) Burn the documents
answer
C. Confidential patient information should be shredded. It is generally collected in large secure containers and shredded at scheduled times.
question
A 52-year-old woman is admitted with dyspnea and discomfort in her left chest with deep breaths. She has smoked for 35 years and recently lost over 10 pounds. Her vital signs on admission are: HR 112, BP 138/82, RR 22, tympanic temperature 36.8°C (98.2°F), and oxygen saturation 94%. She is receiving oxygen at 2 L via a nasal cannula. Which vital sign reflects a positive outcome of the oxygen therapy? A) Temperature: 37°C (98.6°F) B) Radial pulse: 112 C) Respiratory rate: 24 D) Oxygen saturation: 96% E) Blood pressure: 134/78
answer
D. Oxygen therapy increases oxygen saturation. Temperature is not affected by the oxygen. There is no change in heart rate. Administering oxygen should decrease the respiratory rate. The decline in blood pressure is unlikely to be caused by oxygen.
question
The licensed practice nurse (LPN) provides you with the change-of-shift vital signs on four of your patients. Which patient do you need to assess first? A) 84-year-old man recently admitted with pneumonia, RR 28, SpO2 89% B) 54-year-old woman admitted after surgery for fractured arm, BP 160/86 mm Hg, HR 72 C) 63-year-old man with venous ulcers from diabetes, temperature 37.3°C (99.1°F), HR 84 D) 77-year-old woman with left mastectomy 2 days ago, RR 22, BP 148/62
answer
A. Oxygen saturation is low, indicating a problem with ventilation or diffusion, which is related to the respiratory rate.
question
A 56-year-old patient with diabetes admitted for community acquired pneumonia has a temperature of 38.2°C (100.8°F) via the temporal artery. Which additional assessment data are needed in planning interventions for the patient's infection? (Select all that apply.) A) Heart rate B) Presence of diaphoresis C) Smoking history D) Respiratory rate E) Recent bowel movement F) Blood pressure in right arm G) Patient's normal temperature H) Blood pressure in distal extremity
answer
A,B,D,G. You need to determine the patient's usual temperature to evaluate the degree of temperature elevation. Heart rate and respiratory rate increase with temperature. The presence of diaphoresis may contribute to fluid volume deficit from hyperthermia.
question
A 55-year-old widowed patient was in a motor vehicle accident and is admitted to a surgical unit after repair of a fractured left arm and left leg. She also has a laceration on her forehead. An intravenous (IV) line is infusing in the right antecubital fossa, and pneumatic compression stockings are on the right lower leg. She is receiving oxygen via a simple face mask. What sites do you instruct the nursing assistant to use for obtaining the patient's blood pressure and temperature? A) Right antecubital and tympanic membrane B) Right popliteal and right axillae C) Left antecubital and oral D) Left popliteal and temporal artery
answer
B. The only extremity that does not have a compromised artery to auscultate is the right lower leg after the sequential device is removed. The tympanic membrane and temporal artery are affected by facial surgery and oxygen mask.
question
A patient has been transferred to your unit from the respiratory intensive care unit, where he has been for the past 2 weeks recovering from pneumonia. He is receiving oxygen via 4 L nasal cannula. His respiratory rate is 26 breaths/min, and his oxygen saturation is 92%. In planning his care, which information is most helpful in determining your priority nursing interventions? A) Activity order B) Medication list C) Baseline vital signs D) Patient's perception of dyspnea
answer
C. Knowledge of baseline vital signs is needed to prioritize care.
question
During a patient's routine annual physical, she tells you that she has noted that her heart feels like it's " racing, " usually in the later morning, early afternoon, or just before she goes to bed. Her radial pulse rate is 68 beats/min and regular; her blood pressure is 134/82 mm Hg. What additional information is helpful in evaluating the patient's racing heart? (Select all that apply.) A) Dietary habits B) Medication list C) Exercise regimen D) Age, weight, and height
answer
A,B. Dietary habits may include caffeine fluids and foods that stimulate heart rate. Medication list may include pharmacological agents that increase or decrease heart rate.
question
You observe a nursing student taking a blood pressure (BP) on a patient. The patient's BP range over the past 24 hours is 132/64 to 126/72 mm Hg. The student used a BP cuff that was too narrow for the patient. Which of the following BP readings made by the student is most likely caused by the incorrect choice of BP cuff? A) 96/40 mm Hg B) 110/66 mm Hg C) 130/70 mm Hg D) 156/82 mm Hg
answer
D.When you use a blood pressure cuff that is too narrow or short, your patient will most likely have a BP reading that is higher than it really is; you will get a false-high reading.
question
As you are obtaining the oxygen saturation on a 19-year-old college student with severe asthma, you note that she has black nail polish on her nails. You remove the polish from one nail, and she asks you why her nail polish had to be removed. Your best reply is: A) Nail polish attracts microorganisms and contaminates the finger sensor. B) Nail polish increases oxygen saturation. C) Nail polish interferes with sensor function. D) Nail polish creates excessive heat in sensor probe.
answer
C. Nail polish reduces light transmission and can alter oxygen saturation measurement.
question
A patient has been hospitalized for the past 48 hours with a fever of unknown origin. His medical record indicates tympanic temperatures of 38.7°C (101.6°F) (0400), 36.6°C (97.9°F) (0800), 36.9°C (98.4°F) (1200), 37.6°C (99.6°F) (1600), and 38.3°C (100.9°F) (2000). How would you describe this pattern of temperature measurements? A) Usual range of circadian rhythm measurements B) Sustained fever pattern C) Intermittent fever pattern D) Resolving fever pattern
answer
C. The pattern returns to acceptable levels at least once in 24 hours interspersed with fever spikes.
question
A patient presents in the clinic with dizziness and fatigue. The nursing assistant reports a very slow radial pulse of 44. What is your priority intervention? A) Request that the nursing assistant repeat the pulse check B) Call for a stat electrocardiogram (ECG) C) Assess the patient's apical pulse and evidence of a pulse deficit D) Prepare to administer cardiac-stimulating medications
answer
C. Your priority is to assess the patient first. The nurse cannot delegate vital signs to an unstable patient. Therefore first you determine if the patient has a pulse deficit. Calling for a stat electrocardiogram and preparing to administer cardiac-stimulating medications require notification of the health care provider and occur after you assess the patient.
question
Which of the following patients is most at risk for tachycardia? A) A healthy professional tennis player B) A patient admitted with hypothermia C) A patient with a fever of 39.4°C (103°F) D) A 90-year-old male taking beta blockers
answer
C. Patients with a fever have a high heart rate. A healthy athlete has a low heart rate because of conditioning. Hypothermia slows the heart. Beta-blockers reduce heart rate.
question
Which of the following patients is at most risk for tachypnea? (Select all that apply.) A) Patient just admitted with four rib fractures B) Woman who is 9 months' pregnant C) Adult who has consumed alcoholic beverages D) Adolescent awaking from sleep
answer
A,B. Rib fractures would cause splinting and pain to increase respiratory rate. Pregnancy impedes diaphragmatic excursion, causing shallow, frequent breaths.
question
The following blood pressures, taken 6 months apart, were recorded from patients screened by the nurse at the assisted living facility. Which patient should be referred to the healthcare provider for hypertension evaluation? A) 120/80, 118/78, 124/82 B) 128/84, 124/86, 128/88 C) 148/82, 148/78, 134/86 D) 154/78, 118/76, 126/84
answer
C.The definition of hypertension requires two elevated blood pressure measurements in a row. All of the other choices describe prehypertension
question
A patient is admitted for dehydration caused by pneumonia and shortness of breath. He has a history of heart disease and cardiac dysrhythmias. The nursing assistant tells you his admitting vital signs. Which measurement should you reassess? (Select all that apply.) A) Right arm BP: 120/80 B) Radial pulse rate: 72 and irregular C) Temporal temperature: 37.4°C (99.3°F) D) Respiratory rate: 28 E) Oxygen saturation: 99%
answer
B,D,E. An irregular pulse may be the result of a complication of heart disease and requires the assessment of the apical rate. A respiratory rate of 28 is abnormal, yet the oxygen saturation is normal. Both oxygen saturation and respiratory rate would be expected to be outside of the acceptable range.
question
A patient returns to your postoperative unit following surgery for right shoulder rotator cuff repair. The licensed practical nurse (LPN) reports that she had difficulty obtaining the patient's heart rate from his right radial pulse. What is your best response? A) Assess the patient's apical pulse to obtain the heart rate. B) Obtain the heart rate from right and left radial sites. C) Obtain the heart rate using the oximeter probe. D) Perform a complete assessment of all pulses.
answer
D. When an LPN reports that one pulse is difficult to obtain, first you need to assess the patient yourself and compare the quality of all pulses.
question
The nurse prepares to conduct a general survey on an adult patient. Which assessment is performed first while the nurse initiates the nurse-patient relationship? A) Appearance and behavior B) Measurement of vital signs C) Observing specific body systems D) Conducting a detailed health history
answer
A. The nurse inspects appearance and behavior first as part of the nursing assessment. As the patient enters the room, the nurse can observe his or her appearance and behavior, noting any unusual choice of clothing or hygiene or any signs of confusion, anxiety, or happiness.
question
The nurse is teaching a young mother to palpate her 8-year-old child to quickly evaluate if the child has a fever. Which information is important for the nurse to include? A) Place the palm of the hand on the child's back. B) Lightly touch the child's forehead with the fingertips. C) Place the back of your hand against the child's forehead and then on the back of the neck. D) Use the pads of your fingers and press against the child's neck and over the thorax.
answer
C. Temperature is best evaluated by palpating the skin with the dorsum or back of the hand. It is best to select two areas to compare to allow you to detect a change in body surface temperature.
question
While assessing the adult patient's lungs, the nurse identifies the following assessment findings. Which finding should be reported to the health care provider? A) Respiratory rate: 14 B) Pain reported when palpating posterior lower thorax C) Thorax rising and falling symmetrically for right and left lungs D) Vesicular breath sounds heard with auscultation of peripheral lung fields
answer
B. Any areas of tenderness or pain over the posterior thorax could indicate injury such as a broken rib or disturbance of the integumentary system. Further palpation should be avoided until more assessment data are collected, either through further health history or diagnostic testing. All other findings are normal.
question
The nurse is teaching a young female patient to practice good skin health. Which information is important for the nurse to include? A) Avoid sunbathing between 3 PM and 7 PM. B) Oral contraceptives and antiinflammatories make the skin more sensitive to the sun. C) Call the health care provider for the presence of a mole on an arm or leg that appears uniformly brown. D) Wear sunscreen with an SPF of 30 or greater if using a sunlamp or tanning parlor
answer
B, Some medications such as oral contraceptives or antiinflammatory medications may increase the skin's sensitivity to ultraviolet (UV) rays. Skin self-care and self-evaluation practices include avoiding the sun when UV rays are strongest (10 AM to 4 PM). In addition, good skin practices indicate that skin protection should be used when using a tanning bed or sunlamp. Moles that are uniformly brown are not a cause of concern.
question
As a nurse prepares to provide morning care and treatments, it is important to question a patient about a latex allergy before which intervention? (Select all that apply.) A) Applying adhesive tape to anchor a nasogastric tube B) Inserting a rubber Foley catheter into the patient's bladder C) Providing oral hygiene using a standard toothbrush and toothpaste D) Giving an injection using plastic syringes with rubbercoated plungers E) Applying a transparent wound dressing
answer
A,B,D. Adhesive tape, rubber foley catheters, and rubber-coated plungers should be avoided for patients with latex allergies since they can trigger an allergic or anaphylactic response.
question
The nurse is assessing a patient who returned 3 hours ago from a cardiac catheterization, during which the large catheter was inserted into the patient's femoral artery in the right groin. Which assessment finding would require immediate follow-up? A) Palpation of a femoral pulse with a heart rate of 76 B) Auscultation of a heart murmur over the left thorax C) Identification of mild bruising at the catheter insertion site D) Palpation of a right dorsalis pedis pulse with strength of +1
answer
D. A weak pulse may indicate disruption of arterial flow and should be reported immediately. Mild bruising is normal, but if it increases in size, the femoral artery may be leaking, requiring further follow-up with the health care provider. Other findings are within normal limits and do not require notification.
question
The patient reports having a sore throat, coughing, and sneezing. While performing a focused assessment, which finding supports the patient's reported symptoms related to upper respiratory infection? A) Buccal mucosa is moist and dark pink. B) Respiratory rate is 18, rhythm is even. C) Retropharyngeal lymph nodes are enlarged and firm. D) Inspection with a tongue depressor on the posterior tongue causes gagging.
answer
C.The retropharyngeal nodes are located posteriorly to the throat and are enlarged when an infection is located in the throat or pharynx.
question
The nurse is teaching a patient with poor arterial circulation about checking blood flow in the legs. Which information should the nurse include? (Select all that apply.) A) A normal pulse on the top of the foot indicates adequate blood flow to the foot. B) To locate the dorsalis pedis pulse, take the fingers and palpate behind the knee C) When there is poor arterial blood flow, the leg is generally warm to the touch. D) Loss of hair on the lower leg indicates a long-term problem with arterial blood flow.
answer
A,D. A normal dorsalis pedis indicates good arterial blood flow to the lower extremities. Chronic loss of arterial flow results in a lack of hair growth and the appearance of shiny tissue. The dorsalis pedis is located along the top of the foot between the great toe and first toe. When there is poor arterial flow, the skin will be cool.
question
How should the patient be positioned to best palpate for lumps or tumors during an examination of the right breast? A) Supine with both arms overhead with palms upward B) Sitting with hands clasped just above the umbilicus C) Supine with the right arm abducted and hand under the head and neck D) Lying on the right side, adducting the right arm on the side of the body
answer
C. Lying on the back allows breast tissue to relax; raising the arm over the patient's head causes the breast tissue to flatten, and palpation can more accurately locate any nodules or tumors, especially cancerous tumors that are fixed against the chest wall.
question
The nurse is planning a staff education conference about abdominal assessment. Which point is important for the nurse to include? A) The aorta can be felt using deep palpation in the upper abdomen near the midline. B) The patient should be sitting to best determine the contour and shape of the abdomen. C) Always wear gloves when palpating the skin on the patient's abdomen. D) Avoid palpating the abdomen if the patient reports any discomfort or feelings of fullness.
answer
A. Complete abdominal assessment includes inspection, followed by auscultation, palpation, and percussion (if warranted). Anatomically the aorta is located in the upper abdomen and can be palpated on an average-sized patient. The assessment should be performed when the patient is supine so all assessment techniques can be included. Unless there is an open wound or other abdominal drainage, the aorta should be palpated without gloves to be able to assess skin texture, temperature, and any unusual pulsations. Palpation should be performed routinely, but leave areas of discomfort or pain until last.
question
The nurse is teaching a patient how to perform a testicular self-examination. Which statement by the nurse is correct? A) "The testes are normally round and feel smooth and rubbery." B) "The best time to do a testicular self-examination is before your bath or shower." C) "Perform a testicular self-examination weekly to detect signs of testicular cancer." D) "Since you are over 40 years old, you are in the highest risk group for testicular cancer."
answer
A.Men ages 18 to 24 are in the group at most risk for testicular cancer. Teaching should include normal anatomy. A testicular examination should be planned monthly during a shower since the soap and water ease movement of the fingertips over the skin.
question
The patient is assessed for range of joint movement. He or she is unable to move the right arm above the shoulder. How should the nurse document this finding? A) Patient was not able to flex arm at shoulder. B) Extension of right arm is limited. C) Patient's abduction of right arm was limited to 100 degrees. D) Internal rotation of right arm is limited to less than 90 degrees.
answer
C. Abduction of the arm includes raising the arm away from the side and above the shoulder.
question
The nurse plans to assess the patient's abstract reasoning. Which task should the nurse ask the patient to perform? A) "Tell me where you are." B) "What can you tell me about your illness?" C) "Repeat these numbers back to me: 7...5...8." D) "What does this mean: 'A stitch in time saves nine? ' "
answer
D. Abstract reasoning requires cognitive functioning and the ability to identify relationships between concepts.
question
The nurse teaches a patient about cranial nerves to help explain why the patient's right side of the mouth droops instead of moving up into a smile. What nerve does the nurse explain to the patient? A) VII — Facial B) V — Trigeminal C) XII — Hypoglossal D) XI— Spinal accessory
answer
A. The facial nerve innervates the sensory and motor functions of the face above the brow, the cheeks, and the chin and controls face symmetry and smile
question
The nurse is planning to teach the student nurse how to assess the hydration status of an older adult. Which techniques are appropriate for this situation? (Select all that apply.) A) Inspect the lips and mucous membranes to determine if they are moist. B) Pinch the skin on the back of the hand to see if the skin tents. C) Check the patient's pulse and blood pressure. D) Weigh the patient daily.
answer
A,C,D. By assessing for moisture of the mucous membranes and lips, the nurse can quickly evaluate the patient's hydration status. Weighing a patient shows increases of fluid volume from day to day that could result from cardiac problems. This provides useful information about fluid status over time. Blood pressure can indicate fluid status, but be aware it also can be related to other diseases. Skin on older individuals loses its elasticity, and assessing skin on the dorsum of the hand provides inaccurate data regarding skin turgor.
question
Following a bilateral mastectomy, a 50-year-old patient refuses to eat, discourages visitors, and pays little attention to her appearance. One morning the nurse enters the room to see the patient with her hair combed and makeup applied. Which of the following is the best response from the nurse? A) "What's the special occasion?" B) "You must be feeling better today." C) "This is the first time I have seen you look this good." D) "I see that you've combed your hair and put on makeup."
answer
D, When the nurse uses a matter-of-fact approach and acknowledges a change in the patient's behavior or appearance, it allows the patient to establish its meaning.
question
A patient diagnosed with major depressive disorder has a nursing diagnosis of chronic low self-esteem related to negative view of self. Which of the following would be the most appropriate cognitive intervention by the nurse? A) Promote active socialization with other patients B) Role play to increase assertiveness skills C) Focus on identifying strengths and accomplishments D) Encourage journaling of underlying feelings
answer
C. Focusing on strengths and accomplishments to minimize the emphasis on failures assists the patient to alter distorted and negative thinking. The other interventions are important, but they are not designed to impact thoughts.
question
Several staff members complain about a patient's constant questions such as "Should I have a cup of coffee or a cup of tea?" and "Should I take a shower now or wait until later?" Which interpretation of the patient's behavior helps the nurses provide optimal care? A) Asking questions is attention-seeking behavior. B) Inability to make decisions reflects a self-concept issue. C) Dependence on staff must be stopped immediately. D) Indecisiveness is aimed at testing how the staff reacts.
answer
B. Patients with deficits in self-concept often have difficulty making decisions. It is essential for the nurse to remain accepting of the patient and to support him or her in decision making.
question
A depressed patient is crying and verbalizes feelings of low self-esteem and self-worth such as "I'm such a failure...I can't do anything right." The best nursing response would be to: A) Remain with the patient until he or she stops crying. B) Tell the patient that is not true and that every person has a purpose in life. C) Review recent behaviors or accomplishments that demonstrate skill ability. D) Reassure the patient that you know how he is feeling and that things will get better.
answer
A. Demonstrating acceptance of the patient by supportively sitting with him or her builds a therapeutic nurse-patient relationship. The nurse's presence signals value and allows the patient to explore issues of self-concept and self-esteem.
question
An adult woman is recovering from a mastectomy for breast cancer and is frequently tearful when left alone. The nurse's approach should be based on an understanding of which of the following: A) Patients need support in dealing with the loss of a body part. B) The patient's family should take the lead role in providing support. C) The nurse should explain that breast tissue is not essential to life. D) The patient should focus on the cure of the cancer rather than loss of the breast.
answer
A. The nurse should encourage the patient to talk about the threats to body image, including the meaning of the loss, the reactions of others, and the ways in which the patient is grieving.
question
When caring for an 87-year-old patient, the nurse needs to understand that which of the following most directly influences the patient's current self-concept: A) Attitude and behaviors of relatives providing care B) Caring behaviors of the nurse and health care team C) Level of education, economic status, and living conditions D) Adjustment to role change, loss of loved ones, and physical energy
answer
D. Older adults experience significant challenges to self-concept, including mental and physical changes associated with aging and changes in identity and role following retirement and/or loss of significant others. The adjustment to stressors is most important. The other influences are important but to a lesser degree.
question
A 20-year-old patient diagnosed with an eating disorder has a nursing diagnosis of situational low self-esteem. Which of the following nursing interventions would be best to address self-esteem? A) Offer independent decision-making opportunities B) Review previously successful coping strategies C) Provide a quiet environment with minimal stimuli D) Support a dependent role throughout treatment
answer
A. Offering opportunities for decision making promotes a sense of control, which is essential for promoting independence and enhancing self-esteem. Reviewing successful coping strategies is a priority intervention for the nursing diagnosis of ineffective coping.
question
The nurse asks the patient, "How do you feel about yourself?" The nurse is assessing the patient's: A) Identity. B) Self-esteem. C) Body image. D) Role performance.
answer
B. Self-esteem is how a person feels about himself or herself. Asking open-ended questions about self-esteem is important during the nursing assessment.
question
The nurse can increase a patient's self-awareness through which of the following actions? (Select all that apply.) A) Helping the patient define her problems clearly B) Allowing the patient to openly explore thoughts and feelings C) Reframing the patient's thoughts and feelings in a more positive way D) Have family members assume more responsibility during times of stress
answer
A,B,C. These are designed to promote self-awareness and a positive self-concept. Having family members assume more responsibility during stressful times does not help a patient achieve self-awareness; instead it is important to encourage a patient to assume more self-responsibility.
question
When developing an appropriate outcome for a 15-year-old girl, the nurse considers that a primary developmental task of adolescence is to: A) Form a sense of identity. B) Create intimate relationships. C) Separate from parents and live independently. D) Achieve positive self-esteem through experimentation.
answer
A. Understanding developmental tasks across the life span is essential in designing nursing care. Adolescents are focused on establishing their identity outside the family and should be supported in meeting this developmental task.
question
An appropriate nursing diagnosis for an individual who experiences confusion in the mental picture of his physical appearance is: A) Acute confusion. B) Disturbed body image. C) Chronic low self-esteem. D) Situational low self-esteem.
answer
B. Body image involves attitudes related to the body, including physical appearance, structure, or function. Disturbed body image would be an appropriate nursing diagnosis.
question
In planning nursing care for an 85-year-old male, the most important basic need that must be met is: A) Assurance of sexual intimacy. B) Preservation of self-esteem. C) Expanded socialization. D) Increase in monthly income.
answer
B. Self-esteem is essential for physical and psychological health across the life span
question
Based on knowledge of Erikson's stages of growth and development, the nurse plans her nursing care with the knowledge that old age is primarily focused on: A) Intimacy versus Isolation. B) Autonomy versus Shame and Doubt. C) Generativity versus Self-Absorption. D) Ego Integrity versus Despair.
answer
D. The developmental stage of Ego Integrity Versus Despair (Late 60s to Death) is focused on feeling positive about life and its meaning and providing a legacy for the next generation.
question
The home health nurse is visiting a 90-year-old man who lives with his 89-year-old wife. He is legally blind and is 3 weeks' post right hip replacement. He ambulates with difficulty with a walker. He comments that he is saddened now that his wife has to do more for him and he is doing less for her. Which of the following is the priority nursing diagnosis? A) Self-care deficit, toileting B) Deficient knowledge regarding resources for the visually impaired C) Disturbed body image D) Risk for situational low self-esteem
answer
D. Blindness coupled with difficulty ambulating places him at risk for situational low self-esteem. No doubt he and his wife have adapted to the blindness, but his difficulty with ambulation affects many aspects of his life, including self-esteem. However, this low self-esteem is situational; as his mobility improves, his low self-esteem will also resolve. Nothing in the question suggests that the other answers are true.
question
Based on knowledge of the developmental tasks of Erikson's Industry versus Inferiority, the nurse emphasizes proper technique for use of an inhaler with a 10-year-old boy so he will: A) Increase his self-esteem with mastery of a new skill. B) Accept changes in his appearance and physical endurance. C) Experience success in role transitions and increased responsibilities. D) Appreciate his body appearance and function
answer
A. The developmental stage of Industry Versus Inferiority (ages 8-12) is focused on incorporating feedback from peers and teachers, increasing self-esteem with new skill mastery, and promoting awareness of strengths and limitations.
question
The nurse is providing education on sexually transmitted infections (STIs) to a group of adolescents. The nurse knows that further teaching is needed when one of the adolescents states: A) "A vaccine is available to reduce infection from certain types of human papillomavirus." B) "I should be screened for an STI after I am with a new partner." C) "I know I' m not infected if I don't have any symptoms such as discharge or sores." D) "A viral infection such as herpes or human papillomavirus cannot be treated with antibiotics."
answer
C. Many STIs have few symptoms and are often detected during routine screening. The risk of infection is higher in people who are under the age of 25 and who have multiple sex partners. Viral infections cannot be cured with antibiotics, but medication is available to suppress outbreaks. Bacterial infections can be treated with antibiotics, but the infection can recur with new exposure.
question
A 25-year-old patient is in the emergency department and states that she has had a cough and fever for the past 3 days. While performing a physical assessment, the nurse finds several bruises that are in various stages of healing and suspects that the patient possibly is a victim of sexual abuse. Which of the following is the nurse's first action? A) Refer the patient to a sexual counselor B) Tell the patient about the safe house for women C) Ask the patient to describe how she got the bruises D) Report the abuse immediately to the proper authorities
answer
B. The first action is to educate the patient about available resources in the community to help her develop an escape plan. Reporting the abuse to authorities may put her at increased risk for violence but is legally required.
question
A 26-year-old married woman recently discovered that she is pregnant and is at her first prenatal visit. While assessing the patient, the woman's health nurse practitioner discovers that she has purulent vaginal discharge. The patient states, "It burns when I urinate, and I seem to have to go to the bathroom frequently." Based on these symptoms, the nurse practitioner determines that further follow-up is needed because the patient: A) Should be tested for human immunodeficiency virus (HIV). B) May have a sexually transmitted infection (STI) such as chlamydia. C) Is experiencing normal signs of pregnancy. D) Needs education on proper perineal hygiene.
answer
B. Chlamydia does not cause symptoms in about 75% of women; thus they are often unaware that they have an STI. It often causes genitourinary track infections in men and women. Serious complications can result from untreated STIs in pregnancy such as preterm labor and rupture of membranes and premature delivery of the newborn. Purulent discharge indicates infection and is not an expected finding in pregnancy or from poor hygiene practices
question
A new graduate nurse is working in a rehabilitation center that specializes in the care of patients with spinal cord injuries (SCIs). The new graduate knows that sexual issues are common among patients with SCIs. Which of the following actions enhances the nurse's comfort in discussing sexual issues with the patients? (Select all that apply.) A) Clarifying personal values related to sexuality B) Role playing discussion of sexual concerns with another nurse C) Attending a conference to enhance knowledge about sexuality D) Avoiding a discussion of sexual concerns until after completing new nurse orientation
answer
A,B,C. Nurses often avoid discussing sexual issues with patients because they are uncomfortable, lack knowledge, or have personal values in conflict with the patients. Nurses who have difficulty addressing sexual issues need to seek education and experiences to increase knowledge and explore their personal values.
question
The nurse is gathering a sexual history from a 68-year-old man in a nursing home. It is important for the nurse to keep in mind that: A) Older adults are usually not part of a sexual minority group. B) Older adults sometimes do not reveal intimate details. C) Older men and women lose their interest in sex. D) Older adults in nursing homes do not usually participate in sexual activity.
answer
B. Older adults are sometimes hesitant to reveal information relating to sexual issues because they are embarrassed. Sexual health is sometimes not addressed by the nurse, but it is important to include a sexual history as a routine aspect of assessment to communicate that sexual activity is normal. Studies have shown an increase in sexual dysfunction with aging but no decrease in sexual activity or interest.
question
Certain cultural groups in the United States are disproportionately affected by diseases such as human immunodeficiency virus (HIV) and acquired immunodeficiency syndrome (AIDS). The nurse understands that this is most likely caused by: (Select all that apply.) A) Expectations about behavior by men or women in the culture. B) Higher percentages of lesbian, gay, bisexual, or transgender individuals in the culture. C) Genetic predisposition to the disease in the culture D) Communication patterns and language practiced by the culture.
answer
A,D. Cultural factors such as gender, education, socioeconomic status, religion, language, and values influence the use of the health care system. Populations that are at increased risk for HIV are people who are intravenous drug users, those with hemophilia, and those who practice unprotected sex. Genetic factors often increase risk for certain health problems such as cardiovascular disease or cancers but do not usually increase the risk for HIV since it is a viral infection and not a genetic disease.
question
Since the majority of sexually transmitted infections (STIs) have few if any symptoms, it is important for the nurse to: A) Encourage regular screenings in all sexually active individuals. B) Provide information about contraception options. C) Administer prescribed antibiotics for human papillomavirus (HPV) or genital herpes outbreaks. D) Ask all patients if they are experiencing any symptoms.
answer
A. One of the challenges in reducing the incidence of STIs is that most STIs have few symptoms in males or females. Asymptomatic STIs can be diagnosed during a physical examination with appropriate laboratory tests. Screening after each new sex partner is the most effective method to detect and manage STIs. HPV and herpes are viral infections and cannot be treated with antibiotics
question
Establishing trust and encouraging disclosure about sexuality are often facilitated if the nurse begins by asking the patient: A) How often he or she has sexual intercourse. B) To disrobe in preparation for the physical assessment. C) For permission to discuss sexual issues. D) For specific examples of sexual practices and problems.
answer
C. According to the PLISSIT assessment of sexuality (see Box 34-4), the nurse should first ask for Permission to discuss sexual issues with the patient, followed by open-ended questions to determine the patient's concerns.
question
A 15-year-old girl states that she is having unprotected intercourse with her boyfriend. She asks for more information about birth control methods. The nurse informs the patient that: (Select all that apply.) A) Condoms or diaphragms must be used with each sexual encounter. B) Hormonal methods offer little protection against sexually transmitted infections (STIs). C) Barrier methods offer some protection against STIs. D) Sterilization is an effective option that she should consider.
answer
B,C. The most effective methods are longer-acting methods (such as an intrauterine device [IUD] or hormonal injection), which are not associated with the sexual act itself. Sterilization is the most effective method besides abstinence but would not be a good option for a young woman since it is not easily reversible. Hormonal methods do not provide any barrier against STIs, whereas barrier methods may help reduce the risk.
question
The nurse reviews the health history of a 24-year-old woman who indicates that she has had three new sexual partners since her previous examination 2 years ago. The nurse discusses the need for sexually transmitted infection (STI) screening with the patient even though she denies symptoms or discomfort. The nurse realizes that the most serious complication from untreated STIs in females is: A) Genital discharge and dyspareunia. B) Painful menstrual cycles. C) Infertility and pelvic inflammatory disease. D) Genital warts.
answer
C. STIs can certainly cause discharge, discomfort, and genital warts; however, the most serious complications from untreated bacterial STIs are damage to the reproductive organs and increase in the risk of pelvic inflammatory disease, ectopic pregnancy, and infertility.
question
The nurse is providing education about condom use at a community clinic for older adults. Which of following statements demonstrates that the adults understand correct use of condoms? (Select all that apply.) A) "I can use any kind of lubricant such as lotions or baby oil." B) "Before using the condom, I should check the package for damage or expiration." C) "I need to use a condom to help reduce the risk of sexually transmitted infections." D) "A good place to store condoms is in the bathroom so they don't dry out."
answer
B,C. Older adults sometimes are not familiar with condom use and storage. Teach them to use water-based lubricants because oil-based products contribute to breakage of latex condoms. Condoms need to be stored in a cool, dry location away from sunlight.
question
Which of the following represents a nonjudgmental approach when gathering a sexual health history? A) How do you and your wife/husband feel about intimacy? B) Do you have sex with men, women, or both? C) Are you heterosexual or homosexual? D) What is your sexual orientation?
answer
B. A nonjudgmental attitude facilitates trust and open communication between the nurse and patient. Using terms such as partner versus wife or husband allows the patient to identify his or her sexual preference. The terms gay, lesbian, bisexual, or transgender are preferred over the terms heterosexual or homosexual and are more specific in reference to sexual practices.
question
A 54-year-old male patient who is being seen for an annual physical tells the nurse that he is having difficulty sustaining an erection. The nurse reviews his health history and notes no current health problems except medical treatment for depression. The nurse understands that: A) A personal issue such as this is best addressed by the male physician during the examination. B) Erectile dysfunction affects most men over the age of 50. C) The patient needs to be screened for sexually transmitted infections (STIs). D) Antidepressant medication may be affecting his sexual functioning.
answer
D. Many drugs can affect sexual function. Antidepressants can alter sexual functioning by blocking neurotransmitters. The decision to screen the patient is based on his health history, assessment, and sexual practices.
question
The nurse at a community health center is teaching a group of menopausal women about normal changes in the female sexual response that occur with aging. The nurse knows that the information is understood when one of the women states that: A) It's normal for me to take longer to reach an orgasm. B) I might experience chest pain or shortness of breath during intercourse. C) It's normal for me to lose interest in sexual relationships. D) I won't need to be concerned about contraception or sexually transmitted infections because of my age.
answer
A. Normal changes in the female sexual response include a decrease in sex hormone levels, decrease in vaginal lubrication, longer time to reach orgasm, and longer refractory times. Many factors such as chronic illness, medications, stress, or loss of partner can influence the older adult's sexual activity. Older adults may not be as comfortable using barrier methods such as condoms and therefore are at increased risk for sexually transmitted infections.
question
A school nurse is completing a health history on an adolescent female and notices several body piercings and tattoos. The student tells the nurse that she is planning to get more tattoos and piercings over the summer break. The nurse tells the student piercing and tattoos can: A) Prevent you from being involved in contact sports. B) Only create health problems if they are located in the nipples or genital area. C) Increase your risk for infection at the site and in the body. D) Be a safe and important way of establishing your personality.
answer
C. Studies have shown that adolescents with multiple tattoos and piercings are more likely to engage in high-risk behaviors such as drug use and sex with multiple partners. Piercings and tattoos in any location can increase the risk for localized and systemic infection.
question
An emergency department nurse is caring for a patient who was severely injured in a car accident. The patient's family is in the waiting room. They are crying softly. The nurse sits down next to the family, takes the mother's hand, and says, "I can only imagine how you're feeling. What can I do to help you feel more at peace right now? " In this example the nurse is demonstrating: A) Prayer. B) Presence. C) Coaching. D) Instilling hope.
answer
B. The nurse demonstrates presence in this situation by establishing a therapeutic relationship and being with the mother during a particularly stressful time.
question
A patient states that he does not believe in the existence of God. This patient most likely is an: A) Academic. B) Atheist. C) Agnostic. D) Anarchist
answer
B. Atheists do not believe in the existence of God.
question
As the nurse cares for a patient in an outpatient clinic, the patient states that he recently lost his position as a volunteer coordinator at a local community center. He expresses that he is angry with his former boss and with God. The nurse knows that the priority at this time is to assess the patient's spirituality in relation to his: A) Vocation. B) Life satisfaction. C) Fellowship and community. D) Connectedness with his family and co-workers.
answer
A. Many people express their spirituality through their vocation, which can include volunteer positions.
question
A patient who is hospitalized with heart failure states that she sees her illness as an opportunity and a challenge. Despite her illness, she is still able to see that life is worth living. This is an example of: A) Hope. B) Faith. C) Values. D) Connectedness.
answer
A. Hope refers to an energizing source that is oriented to future goals and outcomes. Faith allows people to have firm beliefs despite lack of physical evidence; in this situation the patient is energized by the future and has hope that it will be positive despite her heart failure.
question
Which of the following statements made by an older adult whose husband recently died most indicates the need for follow-up by the nurse? A) "I planted a tree at church in my husband's honor." B) "I have been unable to talk with my children lately." C) "My friends think that I need to go to a grief support group." D) "I believe that someday I'll meet my husband in heaven."
answer
B. Feelings of connectedness are important for the older adult; this statement indicates that this woman is having difficulty staying connected with her children, who are potentially an important resource for the woman.
question
Which of the following nursing interventions support(s) a healing relationship with a patient? (Select all that apply): A) Praying with the patient B) Giving pain medications before a painful procedure C) Telling a patient that it is time to take a bath before family arrive to visit D) Making the patient's bed following hospital protocol E) Helping a patient see positive aspects related to a chronic illness
answer
A,E. Praying with patients and mobilizing the patient's hope create a healing relationship.
question
A patient expresses the desire to learn how to meditate. What does the nurse need to do first? A) Answer the patient's questions B) Help the patient get into a comfortable position C) Select a teaching environment that is free from distractions D) Encourage the patient to meditate for 10 to 20 minutes 2 times a day
answer
C. A quiet environment without distractions enhances learning and is essential for meditation.
question
An older adult is receiving hospice care. Which nursing intervention(s) help the patient cope with feelings related to death and dying? (Select all that apply.) A) Teaching the patient how to use guided imagery B) Encouraging the family to visit the patient frequently C) Taking the patient's vital signs every time the nurse visits D) Teaching the patient how to manage pain and take pain medications E) Helping the patient put significant photographs in a scrapbook for the family
answer
A,B,E. Guided imagery and encouraging connectedness with family members reduce anxiety and enhance coping.
question
Which of the following questions would best assess a patient's level of connectedness? A) What gives your life meaning? B) Which aspects of your spirituality would you like to discuss right now? C) Who do you consider to be the most important person in your life at this time? D) How do you feel about the accomplishments you've made in your life so far?
answer
C. Connectedness refers to feelings about relationships with self, others, and God; this question asks about connectedness with others.
question
A nurse is using the B-E-L-I-E-F tool to complete a spiritual assessment on a 12-year-old male who has recently been diagnosed with acute lymphocytic leukemia. Which of the following questions would the nurse use to assess the child's involvement in the spiritual community? A) Which church do you attend? B) Which sports do you like to play? C) Are there any foods you cannot eat? D) In which church activities do you participate?
answer
D. This question assesses the child's fellowship and involvement in a spiritual community.
question
A nurse is caring for a patient who refuses to eat until after the sun sets. Which religion does this patient most likely practice? A) Islam B) Sikhism C) Hinduism D) Catholicism
answer
A. Patients who practice Islam do not eat until after the sun goes down during the month of Ramadan.
question
A Catholic patient with diabetes receives the following items on his meal tray on the Friday before Easter. For which of the foods does the nurse offer to substitute? A) Apple sauce B) Cheese and crackers C) Spaghetti with meat sauce D) Tossed salad with ranch dressing
answer
C. Most Catholics do not eat meat on Good Friday.
question
A nurse is working in a health clinic on a Navajo reservation. He or she plans care for the patients knowing which of the following is true? A) The patients may not be on time for their appointments. B) The patients most likely do not trust the doctors and nurses. C) The patients probably are not comfortable if they have to remove their undergarments. D) Terminally ill patients probably want to receive the sacrament, the anointing of the sick.
answer
A. Navajos are not always on time for appointments.
question
A 62-year-old male patient has just been told he has a terminal illness. Which of the following statements supports a nursing diagnosis of spiritual distress related to diagnosis of terminal illness? A) "I have nothing to live for now." B) "What will happen to my wife when I die?" C) "How much longer do I have to live?" D) "I need to go to church and pray for a miracle."
answer
A. Patients with spiritual distress verbalize lack of hope and meaning.
question
Which of the following would be the most appropriate outcome for a patient who has a nursing diagnosis of spiritual distress related to loneliness? A) Encourage the patient to meditate 2 to 3 times a week. B) The patient will set up a time to speak to a close friend in 1 week. C) Encourage the patient to phone his brother and set up a time to go out for dinner. D) The patient will experience greater connections with family members in 2 months.
answer
B. This outcome addresses the patient's nursing diagnosis. The statement "The patient will set up a time to speak to a close friend in 1 week" as well as encouraging the patient to set up a time to go to dinner with his brother are both nursing interventions. The statement "The patient will experience greater connections with family members in 2 months" is a goal.
question
Regarding the request for organ and tissue donation at the time of death, the nurse needs to be aware that: A) Specially educated personnel make requests. B) Requests are usually made by the nurse caring for the patient at the time of death. C) Only patients who have given prior instruction regarding donation become donors. D) Professionals need to be very selective in whom they ask for organ and tissue donation.
answer
A. Individuals specially trained in requesting organ donations facilitate the process. They are skilled in talking compassionately to people who have suffered a tragic, sudden loss and have answers to many questions that people have regarding the donation process.
question
The nurse notes that a woman who recently began cancer treatment appears quiet and withdrawn, states that she does not believe the treatments will make any difference, does not ask about her progress, and missed two chemotherapy sessions. Based on the above assessment data, the nurse gathers more information to consider making which of the following nursing diagnoses? A) Anxiety B) Hopelessness C) Spiritual distress D) Complicated grieving
answer
B. The patient exhibits signs and symptoms of hopelessness. Manifestations of hopelessness include withdrawing, not following through with recommended treatment, and losing confidence that anything she does will be of help.
question
A family member asks a home care nurse what he should do if the patient's serious chronic illness worsens even with increased medical interventions. How does the nurse best begin a conversation about the goals of care at the end of life? A) Encourage the family member to think more positively about the patient's new therapy B) Avoid the discussion because it has to do with medical, not nursing, diagnoses C) Initiate a discussion about advance directives with the patient, family, and health care team D) Begin the discussion by asking the patient to identify his or her beliefs about the goals of care while the family member is present
answer
D. If you ask the patient first what he or she believes is best, you know how to discuss that option in more detail and give realistic ways of reaching that desired goal. Discussing other possible options after the patient's preference helps family members know and understand the patient's wishes.
question
Which of the following nursing actions best reflects sensitivity to cultural differences related to end-of-life care? A) Practice honesty with everyone, telling patients about their illness, even if the news is not good. B) Ask family members if they prefer to help with the care of the body after death. C) Provide postmortem care at the time of death to relieve family members of this difficult job. D) Value patient self-determination, understanding that each person makes his or her own decisions.
answer
B. Giving people options in caregiving allows them to honor their cultural beliefs. Although western health care practices place a high value on honesty, people from some cultural backgrounds regard being told the "truth" as harmful.
question
A young man is diagnosed with a serious, life-changing illness. His conversations during his first 2 days of hospitalization are abrupt, superficial, and unrelated to his illness. What understanding about communication enhances your therapeutic communication with this patient? A) Younger patients are usually less talkative about their diagnosis. B) All patients benefit by talking about their feelings with another person. C) Avoid discussing illness-related topics with quiet patients. D) Remain alert for signals that the patient wants to discuss his illness.
answer
D. Make no presumptions about this patient other than the fact that he is not yet ready to talk about his situation. However, stay alert for a time when he might want to talk to you. Some people do not work through their problems by talking to others.
question
A woman experiences the loss of a very early-term pregnancy. Her friends do not mention the loss, and someone suggests to her that she can "always try again." The woman feels confusion over her sadness and stops talking about it with others. What type of grief response is she most likely experiencing? A) Delayed B) Anticipated C) Exaggerated D) Disenfranchised
answer
D. This woman's friends are not fully acknowledging the value of her pregnancy because of the short length of time the woman was pregnant or because, by comparison, the loss seems less than losing a child after birth. The loss does not seem "legitimate." Thus the woman does not experience sympathy from others and feels disenfranchised.
question
A nurse has the responsibility of managing a deceased patient's post-mortem care. Arrange the steps for post-mortem care in the proper order. 1. Bathe the body of the deceased. 2. Collect any needed specimens. 3. Remove all tubes and indwelling lines. 4. Position the body for family visit/viewing. 5. Speak to the family members about their possible participation. 6. Confirm that request for organ/tissue donation and/or autopsy has been made. 7. Notify a support person (e.g., spiritual care provider, bereavement specialist) for the family. 8. Accurately tag the body, indicating the identity of the deceased and safety issues regarding infection control. 9. Elevate the head of the bed.
answer
6, 9, 2, 5, 7, 3, 1, 4, 8 Positioning the head of the bed first helps prevent pooling of blood in the face during all of the other preparations. Find out if there are medical or legal considerations (specimens, autopsy, or tissue donation) before beginning so you do not have to disrupt your care of the person once you have started. Notify a support person for the family while you make other preparations. Invite the family early so you do not violate any cultural or spiritual rituals by beginning your care too early. Once ready to work with the body, remove drains before bathing the body in the event that there is leakage or soiling of the bed on removal. Arrange the person for viewing and transport to the morgue as the last step.
question
A family member of a recently deceased patient talks casually with the nurse at the time of the patient ' s death and expresses relief that she will not have to visit at the hospital anymore. What theoretical description of grief best applies to this family member? A) Denial B) Anticipatory grief C) Dysfunctional grief D) Yearning and searching
answer
B. If a person has been anticipating a loss for some time, he or she may have already experienced many of the emotions (sadness, shock) commonly associated with death.
question
A self-care goal you set when caring for dying and grieving patients includes: A) Learning not to take losses so seriously. B) Limiting involvement with patients who are grieving. C) Maintaining life balance and reflecting on the meaning of your work. D) Admitting that you are not well suited to care for people who are grieving and asking the charge nurse not to assign you to care for these patients.
answer
C. Maintaining life balance is very important for emotional, spiritual, and physical well-being. Withdrawing or not seeing one's work with grieving people as serious does not help maintain balance but rather may contribute to numbing feelings.
question
A nurse is providing postmortem care. Which action is the priority? A) Locating the patient's clothing B) Providing culturally and religiously sensitive care in body preparation C) Transporting the body to the morgue as soon as possible to prevent body decomposition D) Providing all postmortem care to protect the family of the deceased from having to see the body
answer
B. At the end of life religious and cultural expectations are important for the lasting memories held by the family about the way their loved one's death occurred. Sensitive care contributes to feelings of closure, appropriateness of the death rituals, and fulfilled family obligations.
question
Which approach to helping grieving people is most consistent with postmodern grief theories? A) Help the patient identify the tasks to be accomplished during his or her grief. B) Encourage people to recognize stages of grieving in anticipation of what is to come. C) Listen carefully to a person's story of how his or her grief experience is unfolding. D) Offer general grief timelines to help the person know when a phase will pass.
answer
C. Postmodern grief interventions focus on the uniqueness of the patient's story that unfolds and "writes" itself as the person lives through the experience of loss.
question
A patient who has a serious, life-limiting chronic illness wants to continue to engage in self-care and live as normally as possible. Which of the following nursing responses reflect a helpful understanding of patient self-care at the end of life? A) "Learning to accept that you can't perform some activities anymore will bring you more acceptance and peace." B) "Which activities are most important to you, and how can you continue to do them?" C) "People in your life want to help you with things; allow them to do what they want for you." D) "Spending more of your time resting or reading will conserve your energy."
answer
B. Even seriously ill people want to carry on with life, doing what they can to maintain their identity and purpose. They know best how to regulate their energy and wishes for how to spend their time.
question
The nurse suggests that a patient receive a palliative care consultation for symptom management related to anxiety and increasing pain. A family member asks the nurse if this means that the patient is dying and is now "in hospice." What does the nurse tell the family member about palliative care? (Select all that apply.) A) Hospice and palliative care are the same thing. B) Palliative care is for any patient, any time, any disease, in any setting. C) Palliative care strategies are primarily designed to treat the patient's illness. D) Palliative care interventions relieve the symptoms of illness and treatment.
answer
B,D. Palliative care is not reserved for people who are at the end of life. The goal of palliative care is to help relieve the burdens of illness at any time along the continuum of that illness.
question
You have identified three nursing diagnoses for a patient who is having anxiety and hopelessness as a result of a loss. Which general approach do you take to prioritize the nursing diagnoses? (Select all that apply.) A) Use family members and physician orders as primary resources for prioritizing your actions. B) Address the nursing diagnosis that most affects the medical diagnosis. C) Ask the patient to identify the most distressing symptom and first address that diagnosis. D) Use nursing knowledge to address the problem that is the underlying cause of other diagnoses.
answer
C, D. When you are prioritizing nursing diagnoses, first get the patient's sense of the most important issue. Some patients do not fully understand the physiology or relationship among diagnoses. For example, one patient does not understand that pain contributes to a decreased appetite or depression. Your nursing knowledge along with the patient's perceptions help you determine the diagnosis with the highest priority.
question
Regarding grief in older adults, which understanding helps guide your relationship with an elderly patient? A) Older adults have usually sustained many losses in life, which influence the current loss. B) Older adults with a poor memory experience grief less intensely. C) Older adults generally handle loss better because they have more experience with it. D) Social support is less important because an older adult's circle of friends has become smaller.
answer
A. Older adults have usually sustained more losses because they have lived longer. For people at any age, each loss influences the way one responds to subsequent losses. The loss of a social network makes it more important to find resources and sources of social support for grieving older adults. Sometimes many losses overpower a person's coping resources instead of making him or her stronger.
question
When teaching a patient about the negative feedback response to stress, the nurse includes which of the following to describe the benefits of this stress response? A) Results in neurophysiological response. B) Reduces body temperature C) Causes a person to be hypervigilant D) Reduces level of consciousness to conserve energy.
answer
A. Negative feedback senses an abnormal state such as lowered body temperature and makes an adaptive response such as shivering to generate body heat to return the body to hormonal homeostasis.
question
A nurse observes that a patient whose home life is chaotic with intermittent homelessness, a child with spina bifida, and an abusive spouse appears to be experiencing an allostatic load. As a result, the nurse expects to detect which of the following while assessing the patient? A) Posttraumatic stress disorder B) Rising hormone levels C) Chronic illness D) Return of vital signs to normal
answer
C. An increased allopathic load can result in long-term physiological problems and chronic illness. Posttraumatic stress disorder results from a single traumatic event. Hormone levels rise in the alarm stage. Vital signs return to normal in the resistance stage.
question
A patient who is having difficulty managing his diabetes mellitus responds to the news that his hemoglobin A1C, a measure of blood sugar control over the past 90 days, has increased by saying, "The hemoglobin A1C is wrong. My blood sugar levels have been excellent for the last 6 months." The patient is using the defense mechanism: A) Denial. B) Conversion. C) Dissociation. D) Displacement.
answer
A. Denial is avoiding emotional stress by refusing to consciously acknowledge anything that causes intolerable anxiety. This patient's statements reflect denial about poorly controlled blood sugars.
question
When doing an assessment of a young woman who was in an automobile accident 6 months before, the nurse learns that the woman has vivid images of the crash whenever she hears a loud, sudden noise. The nurse recognizes this as ____________.
answer
Posttraumatic stress disorder (PTSD); PTSD originates with a person's experiencing or witnessing a traumatic event and responding with intense fear or helplessness. The car accident is the traumatic event
question
A grandfather living in Japan worries about his two young grandsons who disappeared after a tsunami. This is an example of: A) A situational crisis. B) A maturational crisis. C) An adventitious crisis. D) A developmental crisis.
answer
C. An adventitious crisis is a type of crisis resulting from a natural disaster such as a tsunami.
question
During the assessment interview of an older woman experiencing a developmental crisis, the nurse asks which of the following questions? A) How is this flood affecting your life? B) Since your husband has died, what have you been doing in the evening when you feel lonely? C) How is having diabetes affecting your life? D) I know this must be hard for you. Let me tell you what might help.
answer
B. A developmental crisis occurs as a person moves through life's stages, including widowhood.
question
The nurse plans care for a 16-year-old male, taking into consideration that stressors experienced most commonly by adolescents include which of the following? A) Loss of autonomy caused by health problems B) Physical appearance, family, friends, and school C) Self-esteem issues, changing family structure D) Search for identity with peer groups and separating from family
answer
D. Search for identity with peer groups and separating from family are stressors most commonly experienced by adolescents. Loss of autonomy caused by health problems applies to the older adult. Physical appearance, family, friends and school apply to children. Self-esteem issues and a changing family structure apply to preadolescents.
question
A child who has been in a house fire comes to the emergency department with her parents. The child and parents are upset and tearful. During the nurse's first assessment for stress the nurse says: A) "Tell me who I can call to help you." B) "Tell me what bothers you the most about this experience." C) "I'll contact someone who can help get you temporary housing." D) "I'll sit with you until other family members can come help you get settled."
answer
B. The patients' appraisal of the crisis is the most important area to address first.
question
When assessing an older adult who is showing symptoms of anxiety, insomnia, anorexia, and mild confusion, one of the first assessments includes which of the following? A) The amount of family support B) A 3-day diet recall C) A thorough physical assessment D) Threats to safety in her home
answer
C. Physical causes for problems need to be discovered before treatment for psychosocial problems can be initiated
question
After a health care provider has informed a patient that he has colon cancer, the nurse enters the room to find the patient gazing out the window in thought. The nurse's first response is which of the following? A) "Don't be sad. People live with cancer every day." B) "Have you thought about how you are going to tell your family?" C) "Would you like for me to sit down with you for a few minutes so you can talk about this?" D) "I know another patient whose colon cancer was cured by surgery
answer
C. Ask the patient if he would like you to sit down for a few minutes so he can talk. Providing an open-ended question and an opportunity for the patient to talk allows the nurse to assess the patient's perception of the situation, which is of utmost importance.
question
A 34-year-old man who is anxious, tearful, and tired from caring for his three young children tells you that he feels depressed and doesn't see how he can go on much longer. Your best response would be which of the following? A) "Are you thinking of suicide?" B) "You've been doing a good job raising your children. You can do it!" C) "Is there someone who can help you?" D) "You have so much to live for."
answer
A. Although this sounds abrupt, the patient usually is relieved that you've broached this issue. For safety reasons it is very important to discuss his suicidal thoughts with the patient.
question
The nurse is evaluating the coping success of a patient experiencing stress from being newly diagnosed with multiple sclerosis and psychomotor impairment. The nurse realizes that the patient is coping successfully when the patient says: A) "I'm going to learn to drive a car so I can be more independent." B) "My sister says she feels better when she goes shopping, so I'll go shopping." C) "I've always felt better when I go for a long walk. I'll do that when I get home." D) "I' m going to attend a support group to learn more about multiple sclerosis."
answer
D, Support groups often benefit people experiencing stress.
question
A patient newly diagnosed with type 2 diabetes says, "My blood sugar was just a little high. I don't have diabetes." The nurse responds: A) "Let's talk about something cheerful." B) "Do other members of your family have diabetes?" C) "I can tell that you feel stressed to learn that you have diabetes." D) With silence.
answer
D. The nurse understands that denial is a defense mechanism that assists in coping with a shock. Therapeutic use of silence gives patients time to process their thoughts.
question
A staff nurse is talking with the nursing supervisor about the stress that she feels on the job. The supervising nurse recognizes that: A) Nurses who feel stress usually pass the stress along to their patients. B) A nurse who feels stress is ineffective as a nurse and should not be working. C) Nurses who talk about feeling stress are unprofessional and should calm down. D) Nurses frequently experience stress with the rapid changes in health care technology and organizational restructuring.
answer
D. Nurses frequently experience stress with the rapid changes in health care technology and organizational restructuring and when the situation seems out of their personal control.
question
A crisis intervention nurse working with a mother whose Down syndrome child has been hospitalized with pneumonia and who has lost her entitlement check while the child is hospitalized can expect the mother to regain stability after how long? A) After 2 weeks when the child's pneumonia begins to improve B) After 6 weeks when she adjusts to the child's respiratory status and reestablishes the entitlement checks C) After 1 month when the child goes home and the mother gets help from a food pantry D) After 6 months when the child is back in school
answer
B. Generally a person resolves the crisis and reaches psychological equilibrium in about 6 weeks.
question
If an infectious disease can be transmitted directly from one person to another, it is a: A) Susceptible host. B) Communicable disease. C) Port of entry to a host. D) Port of exit from the reservoir.
answer
B. When an infectious disease can be transmitted directly from one person to another, it is termed a communicable disease. No vector is necessary for transmission.
question
Which is the most likely means of transmitting infection between patients? A) Exposure to another patient's cough B) Sharing equipment among patients C) Disposing of soiled linen in a shared linen bag D) Contact with a health care worker's hands
answer
D. Hands become contaminated through contact with the patient and the environment and serve as an effective vector of transmission.
question
Identify the interval when a patient progresses from nonspecific signs to manifesting signs and symptoms specific to a type of infection. A) Illness stage B) Convalescence C) Prodromal stage D) Incubation period
answer
C. The prodromal stage is the interval between entrance of a pathogen into the body and appearance of first symptoms.
question
Which of the following is the most effective way to break the chain of infection? A) Hand hygiene B) Wearing gloves C) Placing patients in isolation D) Providing private rooms for patients
answer
A. Hands become contaminated through contact with the patient's environment. Clean hands interrupt the transmission of microorganisms
question
A family member is providing care to a loved one who has an infected leg wound. What would you instruct the family member to do after providing care and handling contaminated equipment or organic material? A) Wear gloves before eating or handling food. B) Place any soiled materials into a bag and double bag it. C) Have the family member check with the doctor about need for immunization. D) Perform hand hygiene after care and/or handling contaminated equipment or material.
answer
D. Clean hands interrupt the transmission of microorganisms from family members.
question
A patient is isolated for pulmonary tuberculosis. The nurse notes that the patient seems to be angry, but he knows that this is a normal response to isolation. Which is the best intervention? A) Provide a dark, quiet room to calm the patient. B) Reduce the level of precautions to keep the patient from becoming angry. C) Explain the reasons for isolation procedures and provide meaningful stimulation. D) Limit family and other caregiver visits to reduce the risk of spreading the infection.
answer
C. Patients on isolation precautions may interpret the needed restrictions as a sign of rejection by the health care worker.
question
The nurse wears a gown when: A) The patient's hygiene is poor. B) The nurse is assisting with medication administration. C) The patient has acquired immunodeficiency syndrome (AIDS) or hepatitis. D) Blood or body fluids may get on the nurse's clothing from a task that he or she plans to perform.
answer
D. The gown serves as a barrier between the patient's blood and/or body fluid and potential contact with the caregiver's skin
question
The nurse has redressed a patient's wound and now plans to administer a medication to the patient. Which is the correct infection control procedure? A) Leave the gloves on to administer the medication. B) Remove gloves and administer the medication. C) Remove gloves and perform hand hygiene before administering the medication. D) Leave the medication on the bedside table to avoid having to remove gloves before leaving the patient's room.
answer
C. Gloves need to be changed, and hand hygiene performed to prevent transfer of microorganisms from one source (wound) to another (nurse's hands).
question
When a nurse is performing surgical hand asepsis, the nurse must keep hands: A) Below elbows. B) Above elbows. C) At a 45-degree angle. D) In a comfortable position
answer
B. Keeping hands above the elbows when performing a surgical scrub prevents contaminated water from contact with hands.
question
What is the best method to sterilize a straight urinary catheter and suction tube in the home setting? A) Use an autoclave. B) Use boiling water. C) Use ethylene oxide gas. D) Use chemicals for disinfection.
answer
B. The best sterilizer in a home setting is boiling water.
question
A patient has an indwelling urinary catheter. Why does an indwelling urinary catheter present a risk for urinary tract infection? A) It keeps an incontinent patient's skin dry. B) It can get caught in the linens or equipment. C) It obstructs the normal flushing action of urine flow. D) It allows the patient to remain hydrated without having to urinate.
answer
C. The presence of a catheter in the urethra breaches the natural defenses of the body. Reflux of microorganisms up the catheter lumen from the drainage bag or backflow of urine in the tubing increases the risk of infection.
question
Put the following steps for removal of protective barriers after leaving an isolation room in order: A) Untie top, then bottom mask strings and remove from face. B) Untie waist and neck strings of gown. Allow gown to fall from shoulders and discard. Remove gown, rolling it onto itself without touching the contaminated side. C) Remove gloves. D) Remove eyewear or goggles. E) Perform hand hygiene.
answer
A,B,C,D,E. This sequence ensures that the risk of contamination to other surfaces or health care personnel is minimized.
question
Your ungloved hands come in contact with the drainage from your patient's wound. What is the correct method to clean your hands? A) Wash them with soap and water. B) Use an alcohol-based hand cleaner. C) Rinse them and use the alcohol-based hand cleaner. D) Wipe them with a paper towel.
answer
A. Physically removing wound drainage is most effectively accomplished by washing with soap and water.
question
A patient's surgical wound has become swollen, red, and tender. You note that the patient has a new fever and leukocytosis. What is the best immediate intervention? A) Notify the health care provider and use surgical technique to change the dressing. B) Reassure the patient and recheck the wound later. C) Notify the health care provider and support the patient's fluid and nutritional needs. D) Alert the patient and caregivers to the presence of an infection to ensure care after discharge.
answer
C. Early intervention can reduce the risk of sepsis caused by the progression of the infection. Fever depletes body fluid stores, resulting in an increased risk of dehydration, and providing proper nutrition promotes healing.
question
While preparing to do a sterile dressing change, a nurse accidentally sneezes over the sterile field that is on the over-the-bed table. Which of the following principles of surgical asepsis, if any, has the nurse violated? A) When a sterile field comes in contact with a wet surface, the sterile field is contaminated by capillary action. B) Fluid flows in the direction of gravity. C) A sterile field becomes contaminated by prolonged exposure to air. D) None of the principles were violated.
answer
C. Avoid activities that create air currents, such as sneezing. When you sneeze, microorganisms travel through the air by droplets, contaminating the sterile field.
question
The nurse is giving information to nursing assistive personnel regarding one of the patients who has frequent seizures. What instructions are most important for the nurse to include? A. If a seizure occurs, turn the patient on his side after placing an airway in his mouth B. Hold the patient firmly during the seizure so he does not bump his head C. Pad the siderails of the patient's bed to provide a cushion in case of seizures D. If a seizure occurs, stay with him while watching his ability to breathe
answer
D. Never leaving a seizing patient alone. Loss of the airway and safety are primary issues when a patient has a seizure. Airways are not placed by nursing assistive personnel. He needs to be watched so he does not bump his head, but he should not be held firmly during a seizure. Padding the siderails is not always done, and the question is focusing on when he does have a seizure, which could be anywhere on the unit. Page 341
question
An elderly patient, with a seizure disorder, wears dentures. If the patient has a seizure, when should the nurse remove the dentures? A. When the patient experiences an aura, right before the seizure begins B. During the most active part of the seizure so the airway stays open C. As soon as the seizure is over D. During the postictal period if the dentures are loose
answer
D. If the dentures are loose, then they can be removed after the seizure has ended. Never try to put anything into the mouth or remove anything from the mouth when a patient is having a seizure. If the dentures are in place, leave them alone. Page 344
question
A patient is found unconscious on the floor and may have sustained a head injury. Which immediate action should be implemented by the nurse to prevent further injury? A. Perform a complete head-to-toe assessment B. Place a cervical collar on the patient C. Use at least three people to put him back in bed D. Contact the physician for further orders
answer
B. If a head or neck injury is suspected, then a cervical collar should be placed on the patient to prevent additional injury. It is the only answer that protects the patient from further injury. Using three people would protect the caregivers from injury. Page 344
question
A nurse restrains a violent patient using soft extremity restraints following hospital policy. Nursing care would be correct if which instructions were given by the nurse to nursing assistive personnel? A. "Please explain to the patient why the restraints are needed." B. "Let me know if the skin under the restraints is becoming red." C. "If the patient promises to behave appropriately, release the restraints." D. "If the patient continues to be combative, we can add a vest restraint."
answer
B. Nursing assistive personnel may check for skin changes as a result of the use of restraints as well as circulation of the extremity used. Nursing assistive personnel cannot determine the need for restraints or discontinuing their use. Vest restraints are often not used because of the injury that has occurred with their use. Page 336
question
A patient with a head injury and unpredictable behavior has soft restraints applied to each extremity. Which method by the nurse is most appropriate when removing the restraints for assessment and skin care? A. Remove one hand and one leg restraint at the same time B. Remove the upper extremity restraints at the same time, then the lower ones C. Remove one restraint at a time with another staff member present D. Remove all of the restraints at the same time with another staff member present
answer
C. For safety's sake, one restraint should be removed at a time with an additional staff member present because of the erratic patient behavior. Page 339
question
A patient asks the nurse why his mouth is so dry after receiving radiation therapy to his head and neck. What is the most appropriate response from the nurse? A. "The radiation reduces the flow of saliva." B. "The radiation causes the mucous lining of the mouth to become thin." C. "It would be best to discuss this with your radiation oncologist." D. "How have you been managing the dryness you're having?"
answer
A. Radiation therapy to the head and neck reduces salivary flow and lowers the pH of the saliva, which can lead to stomatitis and tooth decay. Whenever a patient asks a question, the nurse should answer whenever possible and appropriate. Page 436
question
A patient with stomatitis as a result of chemotherapy calls the physician's office to talk with the nurse. What suggestions should the nurse make to help the patient's stomatitis? A. Rinse two to three times a day with an alcohol-based mouthwash and use a water- based mouth moisturizer afterward. B. Rinse your mouth before and after meals and at bedtime with salt water and apply a moisturizing gel on the lips if needed. C. Brush your teeth gently before each meal with water and suck on mints to help soothe your mouth. D. Floss your teeth gently each day after applying an oral lubricant to the mouth and use a Vaseline-based lip gel as needed.
answer
B. This procedure is correct. Alcohol can dry and irritate further the already ulcerated mucous membranes. Page 444
question
Before the nursing assistive personnel (NAP) begin providing hygienic care, the nurse gives them a brief report about their patients. What information is most important for the nurse to share with the NAP? A. Whether any patients have diabetes B. Whether any patients have a tendency to bleed C. Whether any patients need assistance with walking D. Whether any patients have family members present
answer
B. It is essential that staff who will be shaving patients know if bleeding problems exist so the correct equipment for this activity can be obtained. Page 446
question
A diabetic patient recovering from back surgery has long toenails that need cutting both for the patient's comfort and for hygienic purposes. What action should the nurse take to resolve the patient's problem? A. Trim the toenails after soaking them for 10 minutes. B. Ask the physician for a referral to a podiatrist. C. Let the patient trim his own toenails. D. Ask the patient's wife if she will trim his toenails.
answer
B. Patients with diabetes are prone to decreased circulation and increased infection in the lower extremities. It is best to have a professional trim the toenails. The patient would not be able to do it because of his recent back surgery. Page 455
question
The nursing assistive personnel have finished their orientation class covering hygienic care and bed making. Additional teaching is indicated if which action is observed by the nurse while nursing assistive personnel are delivering care? A. A horizontal fold was placed in the linens near the patient's feet while making an occupied bed. B. The patient's eyes were wiped from the outer to the inner canthus. C. The perineum was cleaned from the front to the back. D. The nursing assistive personnel put on gloves before washing a patient's hair.
answer
B. Eyes should be cleaned from inner to outer canthus using a separate part of the washcloth for each wipe. This prevents contamination of the opposite eye if a problem exists. Page 429
question
Two assistive personnel ask the nurse for assistance to transfer a patient from the bed to a stretcher using the three-person lift technique. What is the most appropriate response from the nurse? A. "As long as we use proper body mechanics, no one will get hurt." B. "Because the patient weighs only 100 pounds, you can handle the transfer yourselves." C. "Please find the slide board for us to use." D. "Which one of you wants to be at the patient's head?" A patient with a proprioceptive disorder is being assessed for his ability to walk. Which nursing diagnosis would be the primary one on which to base his care? A. Activity intolerance B. Risk for injury C. Chronic pain D. Nutrition: less than body requirements
answer
B. A problem with proprioceptive function interferes with a person's awareness of his posture and changes in balance. Safety issues would be of primary concern during his assessment as well as care. Page 206
question
The nurse is preparing to stand the patient as part of transferring him to a chair. Which action by the nurse will best facilitate the procedure? A. Rocking the patient back and forth before standing B. Flexing the patient's hips to a 90-degree angle C. Moving the patient to the edge of the bed D. Placing the patient's hands around his neck
answer
A. Rocking the patient gives the patient's body momentum and requires less energy and muscular effort on the part of the nurse to stand the patient. The patient should never place his hands around the nurse's neck. Page 210
question
During an office visit to the physician, an elderly patient admits she has been using her antiembolism stockings less frequently than she did in the past. What response by the nurse is most appropriate? A. "Why haven't you been using the stockings?" B. "You know you're subject to developing blood clots." C. "How many pairs of stockings do you have?" D. "Have you noticed a change in your hands?"
answer
D. Elderly patients may have changes in their hands from arthritis or weakness and may not be able to continue putting on the antiembolism stockings. The use of "why" puts the patient on the defense and is not therapeutic. It is gentler to tell a patient that the stockings improve circulation rather than using the term "blood clots." Page 245
question
A patient who is going to be using crutches asks about wearing shoes with varying sized heels. What statement by the nurse is most accurate? A. "You should wear shoes with the same size heel." B. "Try to wear the same shoes for stability and safety." C. "If you change heel sizes, the crutches may need adjusting." D. "Wearing shoes with different heel sizes is fine as long as you're comfortable."
answer
C. By changing the heel size, the patient's height will be either higher or lower than when the height of the crutches was initially adjusted. Two to three fingers should fit between the axilla and the crutch, and the elbow should be slightly bent for proper fit. Page 248
question
A patient with a weak right leg uses a cane. Where should the nurse stand while ambulating this patient? A. On the patient's left side B. On the patient's right side C. Slightly in front of the patient and on the patient's left side D. Slightly behind the patient's right side
answer
B. When a patient is using a supportive device such as a cane, the nurse stands on the weak side, which in this case would be the right side. The patient's cane would be in the left hand, because the weakness is on the right. Page 249
question
A patient is being taught how to get up from the chair to the walker. Instructions by the nurse would be correct if which statement were included? A. "Use the walker's handgrips to give you leverage." B. "Get close to the edge of the chair." C. "Rock yourself several times to get up." D. "Use the arms of the chair to push up to the walker."
answer
D. The arms of the chair are more stable to use when standing. The walker could tip over. Rocking the patient works when another person is helping stand a patient. Sitting close to the edge of the chair is helpful anytime someone is getting up; it is not specific to this question. Page 258
question
A patient using a CPM machine complains of increasing pain on the heel of the affected leg. Which action by the nurse should be done first? A. Check the functioning of the CPM machine. B. Perform skin care on the affected extremity. C. Pad the hard surfaces of the CPM machine. D. Assess the skin on both lower extremities.
answer
D. The patient should always be assessed first, before checking equipment or implementing any treatment. Page 239
question
A patient complains of spongy, bleeding and unusually red gums during a routine office visit. The nurse would ask the patient about his daily intake of which nutrient? A. Protein B. Vitamin C C. Iron D. Vitamin B12
answer
B. Patients with vitamin C deficiencies manifest the signs and symptoms listed. None of the other nutrients listed cause these problems. Page 811
question
A patient has a body mass index of 38. What topic would the nurse expect to discuss with the patient? A. The foods to eat to reduce his blood glucose levels B. The need to increase his daily protein intake C. The health risks related to extreme obesity D. The need to add a few pounds, but slowly
answer
C. When a person has a body mass index of 30 or above, the person is considered obese. It is appropriate to discuss the risks of carrying the extra weight. Page 813
question
An alert, oriented patient is recovering from back surgery and must remain flat, although he may turn on his side. What approach should be used by the nurse to meet his nutritional and safety needs during meal time? A. Open food and beverage containers, cut food and position tray so he can reach it B. Prepare the items on the tray, then watch the patient's ability to feed himself and assist as needed C. Leave the tray where the patient can reach it, which allows the patient to be totally independent D. Sit with the patient and feed him after opening and preparing the food on the tray
answer
B. The patient should be allowed to do as much as possible, but the nurse needs to be nearby and assist as needed. This allows the patient as much independence as possible. The other options can cause the patient difficulty in eating the meal, or the patient may not want to be fed. Page 817-819
question
A patient requires a "dysphagia advanced" diet. Which snack would be most appropriate for the nurse to bring to the patient? A. A jelly sandwich B. Graham crackers with peanut butter C. Pretzels D. Apple slices
answer
A. The jelly sandwich. Patients in this group cannot have very hard, sticky, or crunchy foods. All of the other snacks listed are either crunchy or hard. Page 823
question
The nurse has patients from a variety of religious and cultural groups. The nurse would need to intervene if which meal was served? A. A Hindu patient receives a baked potato, mixed vegetables, dinner roll, and salad B. An Orthodox Jew received a grilled cheese sandwich, French fries, coleslaw, and applesauce C. A Muslim patient receives baked ham, scalloped potatoes, mixed fruit, and garlic bread D. A Southeast Asian patient receives baked chicken, rice, corn, and a fruit salad,
answer
C. Muslims do not eat pork and ham is from a pig. All of the other meals are correct. Hindus are generally vegetarian out of respect for life. The Orthodox Jew has no meat on the tray, so there is no problem with the food being Kosher. The Asian patient is most likely lactose intolerant, but there are no milk products on the tray. Page 811
question
The nurse is checking placement of a feeding tube of a patient who requires enteral medications. The procedure would be correct if which technique, by the nurse, was observed? A. The nurse pulled back quickly with the 20-mL syringe watching for facial grimacing B. The nurse placed the patient supine with the bed elevated at least 30 degrees while checking placement C. The nurse pulled the syringe plunger slowly to obtain 10 mL of gastric juice at least an hour after medications were given D. The nurse flushed the tube with 60 mL of air through the syringe while watching for abdominal distention
answer
C. The nurse needs to wait at least an hour after medications are given to allow for absorption. The syringe used should be a 60 mL syringe to allow for ample room for both aspiration and irrigation. The patient should be kept with the head of the bed at 30 degrees minimum. A small amount of gastric fluid aspirated slowly can be tested for the appropriate pH level, which would indicate proper technique and proper placement of the tube. Page 836
question
A patient's oxygenation saturation level has gone from 97% to 91% during a coughing spell even though the small-bore feeding tube seems to be in place. What action should the nurse take initially? A. Stop the continual tube feeding B. Elevate the head of the bed C. Auscultate the patient's lungs D. Notify the physician
answer
A. The feeding must be stopped while the patient's status is being evaluated. If the tube has become misplaced, this action prevents additional fluid from being instilled. The nurse would do all of the actions listed and in the order they appear in the options listed. Page 837
question
An elderly patient with a history of constipation requires an enteral tube into the intestinal tract. What assessment by the nurse would be most important before insertion of the tube? A. Auscultating the lungs B. Checking the skin turgor C. Weighing the patient D. Checking for bowel sounds
answer
D, Administration of enteral nutrition is often delayed until bowel sounds are heard. It has been found that feedings might begin before bowel sounds are present, especially in patients with jejunostomy tubes, but this can present a risk. Page 8
question
A patient on enteral nutrition has gained four pounds over the past 48 hours. What action should the nurse initially take? A. Check the current laboratory values B. Check for signs of fluid retention C. Slow the rate of the tube feeding by 20% D. Ask the patient how he feels
answer
B. Slow weight gain is expected. A gain of more than two pounds in 24 hours usually indicates fluid retention. This patient has gained four pounds in 48 hours and is probably retaining fluid. Asking the patient how he feels would provide subjective data and objective data is needed to determine why the patient is gaining weight rapidly. Page 8
question
A patient receiving enteral feedings develops diarrhea and abdominal cramping. What additional information should the nurse check before notifying the physician? A. If the formula or medications contain sorbitol B. If the patient has a history of a latex allergy C. The patency of the enteral feeding tube D. A complete assessment of the gastrointestinal system
answer
A. Sorbitol can cause diarrhea. None of the other options are correct. Page 8
question
A patient with a complex medical condition and an unusual family situation has just been admitted to the unit. Which type of charting would be most appropriate for the nurse to use? A. SBAR documentation B. Charting by exception C. Focus charting D. PIE documentation
answer
A. Because of the complexity of the patient's medical condition and family situation, SBAR documentation provides a framework for cross-discipline communication in an efficient, timely format. Pages 52-53
question
The nurse is documenting the care delivered to his patients. The best documentation would contain which characteristics? A. The majority of the documentation provides subjective data. B. The nurse's hunches are included in case a sudden change occurs in the patient's condition. C. The documentation contains only objective data. D. The documentation reflects individualized care based on assessment data.
answer
D. All nursing care is based on the patient's needs, which are identified during the assessment process. Both objective and subjective data are included, but not the nurse's hunches or personal thoughts. Page 46
question
During report, the nurse includes that a positive variance has occurred with one of the patients. Which information provided by the nurse would support this statement? A. A Foley catheter needed to be inserted because the patient could not void. B. The patient's fever dropped dramatically and sooner than expected. C. The patient had to be taken back to surgery. D. The patient's family has been visiting frequently.
answer
B. The information must be focused on the patient and no one else and would show an improvement in the patient's condition. The other options either show a negative situation or are not focused on the patient. Page 53
question
Unexpected events occur in the health care arena. When would an incident report need to be completed? A. When less than standard patient care has been provided B. To document an injury to a patient or visitor C. To identify potential risks in new treatments D. To document when an adverse situation almost occurred in care
answer
B. Whenever unexpected adverse events result in injury either to caregivers or to the recipients of care, an incident report needs to be completed. Page 61
question
After receiving a narcotic for pain, the patient's respirations drop to a dangerous but stable level. Which documentation statement regarding this situation is best? A. "Too much morphine was given; being monitored frequently and is stable; family at bedside and has been told of situation." B. "Incident report filed after patient received too much pain medication and had decreased respirations; is resting quietly; doctor notified." C. "Dilaudid 1 mg IV caused RR 8, BP 100/68, P 70 afterward; being monitored q15 min—see graphic for VS; MD notified." D. "Sleeping deeply and snoring after receiving narcotic IV; VS stable; nailbeds pink, oxygen ready if needed; supervisor notified."
answer
C. Data should be objective, reflecting the patient's status. Never document in the record that an incident report was completed. Page 47
question
The nurse is assessing the temperature of a confused patient with a fever. Which action by the nurse will help most to provide an accurate measurement? A. Dry the axilla before placing the thermometer probe. B. Hold the thermometer probe in place. C. Place the patient in a supine position. D. Check that the patient has not had anything to eat or drink recently.
answer
A. It is common to find sweat in the axilla of a patient with fever. Drying the axilla helps the probe make better contact against the skin. The axillary route would be best because of the patient's confusion. Regardless of the site used, the probe must be held in the appropriate position for maximum contact. Page 73
question
After checking the tympanic temperature two consecutive times in an 82-year-old patient, the nurse finds the reading to be several degrees lower than expected. What is the most appropriate nursing action at this time? A. Obtain a different thermometer. B. Check for the presence of cerumen. C. Document the temperature assessed. D. Record the average between the two readings.
answer
B. With aging, cerumen (earwax) tends to be drier and can interfere with an accurate tympanic temperature measurement. The cerumen acts as a barrier and therefore the temperature reading would not be reliable. Page 76
question
A patient with abdominal ascites is having his respiratory rate assessed. The technique will be judged as most appropriate if which nursing procedure is observed? A. The patient is assessed while flat, supine, and quiet. B. The patient's head is elevated based on the patient's desire. C. The nurse tells the patient when to begin breathing for the assessment. D. The nurse holds the patient's wrist while counting respirations.
answer
B. With increased fluid in the abdomen, the patient's respiratory status can be compromised. For patient comfort and accuracy of assessment, the patient's head should be elevated. Page 87
question
The nurse is preparing to assess a patient's blood pressure. Which statement by the nurse will help promote an accurate reading? A. "Just relax while I put the cuff on your arm." B. "This is painless and will take just a minute." C. "The cuff can go over your thin silk sleeve." D. "Please uncross your legs while I do this."
answer
D. Crossing the legs can falsely increase the systolic as well as diastolic blood pressure. The cuff should be on the bare arm, but the uncrossing of the legs is more important. The other two statements are true but would not necessarily promote an accurate reading. Page 93
question
A new nurse is learning which patients should not have an electronic blood pressure measurement done. Staff education will be judged as most successful if which patient assigned to the new nurse is assessed using this technique? A. The patient with a constant tremor B. The diabetic patient with hyperglycemia C. The feverish patient who is shivering D. The patient with an irregular heart rate
answer
B. Patients with constant body movement as well as irregular heart rates, peripheral vascular disease, and seizures are not candidates for having the electronic blood pressure used. However, the diabetic patient is an appropriate candidate. Page 99
question
A patient is being taught how to conduct a self-examination of his skin and scalp. Teaching will be judged as effective if the patient tells the nurse he will call the physician's office if he makes which observation? A. A small dark mole looks the same. B. A smooth, flat mole has gotten larger since his weight gain. C. A small black mole is beginning to ooze. D. A raised mole has regular borders.
answer
C. Whenever a change occurs in a mole, it should be brought to the attention of the physician. This is especially important in this situation where the mole is black and is oozing. The smooth, flat mole might enlarge with weight gain. The other moles have normal descriptions. Page 119
question
The nurse is beginning a health assessment of an adult patient. What patient information should the nurse include as part of the general survey? A. Cardiac and respiratory status B. Prior and current health history C. Pain level and current medications D. Overall appearance and behavior
answer
D.Overall appearance and behavior give the nurse a brief idea of whether or not the patient is in distress and that an emergency exists. Page 116
question
A patient with known cardiovascular health issues is having his respiratory system assessed by the nurse. Which approach by the nurse is most appropriate to obtain accurate data? A. Move through the assessment as quickly as possible. B. Ask closed-ended questions to conserve the patient's energy. C. Elevate the head of the patient's bed for comfort. D. Count the respirations for 15 seconds and multiply by 4.
answer
C. When a patient has cardiovascular health problems, the respiratory status may also be affected. It is recommended to elevate the head of the bed for patient comfort. Do not hurry the patient as this can upset the patient and make him anxious, which would not help his cardiovascular problem. A complete health history is needed, and asking closed-ended questions will not obtain the information needed. No shortcuts should be taken when assessing the respirations or any of the vital signs on a patient with a known cardiovascular history. Page 136
question
The nurse is performing a pupil assessment on a patient who is confused and unable to follow directions. Which component of the pupil exam would most likely be inaccurate? A. Accommodation B. Consensual response C. Equality of pupils D. Shape of the pupils
answer
A. Accommodation requires the patient to follow an object with the eyes to determine convergence and divergence as well as constriction and dilation of the pupil depending on whether the object is brought closer or further away from the eyes. The other pupil assessments do not require the patient's cooperation. Page 160
question
The nurse determines that the patient has an irregular heart rate. Which action by the nurse can determine if a pulse deficit is present? A. Count the apical heart rate for 30 seconds, and then the radial pulse for the next 30 seconds. B. Count the apical pulse for 60 seconds, and then wait 5 minutes and count the radial pulse for 60 seconds. C. Count the apical rate for 60 seconds while another nurse counts the radial pulse at the same time. D. Count the radial pulse for the first 30 seconds, and then the apical rate for the last 30 seconds.
answer
C, The only way a pulse deficit can be determined is if both the apical and the radial pulses are assessed at the same time. Page 140
question
The physician changes the oxygen order from 3 L/m to 4 L/m via nasal cannula. What nursing action is indicated as a result of the change in flow rate? A. Assess the patient's respiratory rate more often B. Add humidification to the nasal cannula C. Elevate the patient's head to at least 45 degrees D. Encourage use of the incentive spirometer
answer
B. When the oxygen rate is 4 liters per minute or greater, humidification is needed to prevent drying of the nasal mucous membrane. Page 631
question
A patient with sleep apnea asks the nurse why his physician has ordered "something called CPAP." Which response by the nurse is best? A. "The alveoli need to be kept inflated to provide for better gas exchange, which the mask will do." B. "CPAP uses a nose mask that pushes air in at a constant pressure to keep your airway open." C. "You really need to ask your doctor since she's the one who ordered it." D. "Your oxygen level drops when you're sleeping, and CPAP will keep the level up."
answer
D. This is the clearest answer to the patient's question. Do not use technical language the patient might not understand. Page 642
question
A patient with underlying COPD had BiPAP started 30 minutes ago. The nurse should inform the patient that he will be having which diagnostic test within the hour? A. Arterial blood gases B. A chest radiograph C. A pulmonary function test D. A pulse oximeter reading
answer
A. The arterial blood gases need to be drawn to measure for carbon dioxide retention, which the oxygen saturation level does not provide. The test is invasive and uncomfortable. Page 644
question
A patient is to be placed on a ventilator. Which nursing action has been found to be most effective in reducing ventilator-associated pneumonia? A. Performing mouth care at least four times a day B. Repositioning the patient every 2 to 3 hours C. Assessing lung sounds every 4 hours D. Performing range of motion exercises three times a day
answer
A. Studies have shown that frequent mouth care decreases the incidence of ventilator-associated pneumonia. The other procedures are important to do, but they do not affect the incidence of ventilator-associated pneumonia. Page 651
question
The high-pressure alarm on a patient's ventilator has sounded. The nurse should check to determine if which situation has occurred? A. The patient was coughing B. The patient became disconnected from the ventilator C. There is a leak in the ventilator circuit D. The patient's anxiety is increasing
answer
A. Coughing causes the high-pressure alarm to sound. Options 2 and 3 deal with low-pressure alarms. Page 647
question
A patient's wife asks why that "funny looking bag" was attached to her husband's breathing tube and squeezed before he was suctioned. Which response by the nurse is most appropriate? A. "This gives him extra oxygen before his tube is suctioned." B. "This maneuver compensates for the anticipated suction-induced hypercarbia." C. "What are your concerns regarding the procedure?" D. "It helps prevent the lungs from collapsing while he is being suctioned."
answer
A. This is a simple, truthful statement that would not overwhelm the patient's wife. Option 2 is highly technical. Option 3 does not answer the patient's wife. Option 4 is incorrect. Page 679
question
The nurse is preparing to perform nasopharyngeal suction on a patient. Which assessment is most important for the nurse to do before beginning the procedure? A. Whether the patient has been sick within the past 3 months B. Whether the patient has ever had the procedure done before C. Whether the patient has any environmental allergies such as allergies to dust or grasses D. Whether the patient has an allergy to shellfish
answer
C. Allergies that cause swelling of the mucosa may narrow the nasal passage, which can affect the nurse's ability to easily pass a suction catheter. Options 1 and 2 have no direct correlation with the procedure, and no iodine-based products are used, so the question about allergy to shellfish (Option 4) is unnecessary to ask. Page 669-670, 677
question
A patient has a cuffed fenestrated tracheostomy tube. Which activity would the nurse anticipate has been done before the inner cannula is removed? A. Evaluation of the patient's risk for aspiration by a speech pathologist B. Measurement of the patient's arterial blood gases by respiratory care C. Determination of the status of the patient's lung sounds D. Review of how long the patient has had his artificial airway
answer
A. Speech pathologists evaluate the patient for aspiration risk before cuff deflation. None of the other answers are affected by the cuff deflation. Page 689, 695
question
The nurse has just replaced soiled tracheostomy ties alone on a patient with a cuffed tracheostomy tube. Nursing care would have been appropriate if the nurse used which technique? A. Assessed the skin on the patient's neck for redness and intactness B. Cut the old tracheostomy ties on the side after securing the new ones first C. Had the patient cough and deep breathe before removing the old tie D. Moved the pilot balloon to the side where the tracheostomy ties were cut
answer
B. The new tracheostomy ties must be secured before the old ones are removed if no assistant is present, because dislodgement of the tracheostomy tube could occur. If the pilot balloon is cut, the tracheostomy cuff will deflate, which can cause major problems if the patient is at risk for aspiration or is on mechanical ventilation. The pilot balloon must be away from the area where the tracheostomy ties will be cut. Page 692
question
A patient with a head injury has increased intracranial pressure. What action before the patient's endotracheal tube is suctioned helps to reduce an additional rise in intracranial pressure? A. Elevating the patient's head of the bed to 45 degrees B. Hyperoxygenating the patient C. Placing sterile normal saline into the endotracheal tube D. Checking the patient's oxygen saturation
answer
B. Suctioning can cause a rise in intracranial pressure in patients with head injury. By hyperoxygenating the patient before suctioning, the amount of carbon dioxide is lowered, which induces vasoconstriction. Vasoconstriction reduces the potential for an elevation in intracranial pressure. All the other answers are incorrect. Page 679
question
Several family members want to assist with postmortem care on their loved one. What action by the nurse is most important in this situation? A. Support the family as they experience their grief and loss. B. Before beginning the care, explain the sounds the expired patient may make C. Help the family members put on the appropriate personal protective equipment D. Allow the family members to do only those activities they are comfortable with
answer
C. The family members participating in postmortem care need to be protected from the deceased patient's body fluids. All of the answers are correct; however, only option 3 deals with the physical wellbeing of the family members. All of the other options deal with the psychological aspects. Page 416
question
A dying patient verbalizes that he cannot get comfortable, although his pain level is well managed. Which nursing actions would be most appropriate to meet this patient's needs? A. Reposition the patient after giving him a backrub. B. Turn out all the lights after telling him where he is. C. Suction the patient's mouth, then perform mouth care. D. Raise the head of the bed after changing all the linens.
answer
A. Comfort measures such as a backrub can help relieve pressure and soothe the patient as well as a position change. Page 418
question
An elderly patient has just died and has dentures in place. What action can the nurse take to keep the jaw in normal position? A. Place a rolled towel under the chin B. Tape the jaw closed using paper tape C. Use roll gauze around the head and jaw D. Place a cervical collar on the patient
answer
A. Placing a towel under the chin is the least invasive measure of maintaining the jaw position so the dentures remain in place. The jaw muscles weaken after death and the dentures could move. Check to ensure this is allowed by the beliefs of the family. Page 416
question
A dying patient's respirations are 24, slightly labored, but clear although he's becoming more restless. What nursing intervention would be most helpful? A. Decrease the amount of oral fluid given B. Monitor the oxygen saturation every four hours C. Reposition the patient into semi-Fowler's D. Place the patient on his side with his neck hyperextended
answer
C. The patient's ventilatory efforts need to be supported, and this can best be achieved by elevating the head of the bed, which promotes maximal ventilation, lung expansion and drainage of secretions. The patient's respirations are clear, not noisy, so turning the patient on his side isn't indicated. There is enough information provided to intervene now. Monitoring the oxygen saturation every 4 hours will not do anything to improve the situation now. Page 412
question
A family suggests using a heating pad to warm the feet of a family member who is dying at home. Which response by the nurse is most appropriate? A. "Put socks on and a light blanket over the legs and feet." B. "You can use a heating pad as long as it's on the lowest setting." C. "Do you have an electric blanket instead of a heating pad?" D. "Ask you family member what he wants done."
answer
A. Electrical heating devices should not be used because of the chance of burning the patient because of decreased circulation, which also causes decreased sensation to heat. Page 418
question
The nurse has just removed his gloves after providing care for his patient. What is the next action the nurse should take? A. Document the care provided. B. Check for new orders. C. Restock the patient's supplies. D. Perform hand hygiene.
answer
D. As soon as a procedure or care is completed, hand hygiene is indicated. Page 176
question
A urine specimen has just been collected and placed in the appropriate container. What action by the nurse is indicated next? A. Perform hand hygiene. B. Place the container in the agency's biohazard bag. C. Document the collection in the patient's record. D. Open the patient's curtains.
answer
B. Many agencies have bags in which liquid specimens are placed to prevent contamination. This would be the next step, then performing hand hygiene, and then documenting the collection of the specimen. Page 183
question
The nurse has dry, cracked skin on his hands and is preparing to give patient care. What is the appropriate action for the nurse before beginning the patient's care? A. Check for new orders. B. Place a dab of petroleum-based lotion on his hands. C. Apply clean gloves. D. Perform hand hygiene.
answer
D. Hand hygiene needs to be done before beginning care. Lotion would be indicated after the hands were washed, but not with a petroleum-based lotion. Different products should be tried to minimize cracked skin. Page 179
question
After receiving report, the nurse prepares to give care to her patient who has droplet precautions. Correct isolation technique would be demonstrated if which nursing action were taken? A. The nurse wears a mask while talking to the patient from the doorway. B. The nurse wears a gown and mask when taking supplies into the room. C. The nurse wears a mask while checking the patient's temperature. D. The nurse wears gloves and a gown while moving the patient up in bed.
answer
C. A mask needs to be worn when within 3 feet of a patient on droplet precautions. The nurse would be within 3 feet during temperature assessment. A gown is not necessary for droplet precautions. Page 181
question
The nurse is preparing to remove isolation barriers consisting of gown, gloves, goggles, and mask. Contamination of the nurse will be prevented if the items are removed in which order? A. Mask, goggles, gloves, then gown B. Gloves, goggles, gown, then mask C. Gloves, gown, goggles, then mask D. Gown, gloves, mask, then goggles
answer
B. This technique prevents the nurse from becoming contaminated. Page 184
question
A nurse with a latex allergy needs to perform a sterile procedure and finds the only sterile gloves available are latex. What action by the nurse would be most effective in solving the problem? A. Rub petroleum jelly on his hands to provide a barrier between his hands and the gloves. B. Put a pair of synthetic gloves on before donning the latex sterile gloves. C. Use a larger pair of sterile gloves so they are not as tight. D. Rinse his hands with cold water before putting on the sterile gloves.
answer
B. Placing a pair of synthetic gloves on before the latex gloves provides a barrier between the skin and the latex. Petroleum products break down latex products and should not be used. The other two options are ineffective. Page 199
question
A patient with a urinary catheter inserted 36 hours ago begins to have itching, hives, and a runny nose. What question should the nurse ask first? A. "Are you having any difficulty breathing?" B. "Are you having any chest discomfort?" C. "Do you have any food allergies?" D. "Have you ever felt this way before?"
answer
A. The status of the airway is most important to determine and can give the nurse information about the seriousness of the reaction to latex. All of the questions are pertinent, but the ABCs of CPR would be most important to determine. Page 199
question
The nurse has a cold and needs to change a dressing on a patient who is immunocompromised. Which action by the nurse would be most appropriate? A. Ask another nurse to change the dressing. B. Wear a gown and mask when changing the dressing. C. Perform hand hygiene for a longer time before putting on sterile gloves. D. Ask the patient if it is all right with him if he changes the dressing.
answer
A. When a nurse is sick and needs to get close to an open wound of a patient whose immune system is not functioning well, it is appropriate to ask someone else to change the dressing, rather than put the patient at risk. This is the best answer and the only one that keeps the sick nurse away from the patient. Page 190
question
An elderly, restless patient needs to have a sterile packing replaced in his hip wound. Care would be appropriately delegated if the nurse assigned the nursing assistive personnel (NAP) to perform which activity? A. Position the patient using as many pillows as needed. B. Open the sterile supplies for the nurse to decrease the length of the procedure. C. Give the patient a backrub during the procedure to distract him. D. Place the patient in a position that is comfortable during the dressing change.
answer
D. It is appropriate to have the nursing assistive personnel position the patient in the most comfortable position possible. The patient will need to remain quiet to prevent contamination of the wound site during the sterile dressing change. Page 194
question
A nurse is setting up an over-bed table with supplies to clean a wound and to replace the sterile dressing. The nurse would need to consider that contamination exists if which situation is present? A. The sterile gloves are kept above waist level. B. The over-bed table is positioned before any of the supplies are opened. C. The nurse pours the bottle of cleaning solution with the palm facing the label. D. The sterile glove package is dry and intact but water stained.
answer
D. If a package has a water stain, it is considered contaminated even if the package looks intact and is currently dry. Page 193
Get an explanation on any task
Get unstuck with the help of our AI assistant in seconds
New